Download as pdf or txt
Download as pdf or txt
You are on page 1of 43

31/10/2022, 15:39 Question Paper

Countdown Mock 08 (CLAT) 2023


Directions: Read the passages given below and answer the questions that follow.

Passage – 1

The idea of a Uniform Civil Code (UCC) is once again in the limelight because of the recent observations made by
the Supreme
Court. Besides the desirability of the UCC, these observations also raise issues about the court's
role in questioning the government
on framing and implementing a UCC in a democracy founded on the principle
of separation of powers. I deal with both these aspects
in this article.

This is not the first time that the court has called for implementing Article 44 of the Constitution, enjoining the state
to "endeavour to
secure for the citizens a uniform civil code throughout the territory of India." In Sarla Mudgal
(1995), the court controversially said
that a UCC modelled on Hindu law would resolve the treatment of bigamy in
different communities. Following widespread outrage,
the court clarified that its views were merely observations
and not binding. In Pannalal Bansilal (1996), the court came full circle,
saying that "a uniform law, though is highly
desirable, enactment thereof in one go perhaps may be counter-productive to [the] unity
and integrity of the nation.
The mischief or defect which is most acute can be remedied by process of law at stages." The court will
find it
impossible to reconcile this earlier, and arguably more reasonable, position with its present observations.

This is not to say that a UCC should be indefinitely postponed. However, the goal of Article 44 is perhaps
misunderstood. In Shah
Bano (1985), it was said that a UCC would help national integration by removing disparate
loyalties to conflicting laws

But the UCC is not a magic wand for uniformity, nor will uniformity achieve national integration overnight.Instead, a
UCC ought to
have the objective of reducing gender discrimination by empowering women and restoring their
dignity and self-esteem. Such an
approach can radically alter stakeholder perspectives, and give an impetus of a
different nature.

An interesting phenomenon in this context exists in the subcontinent itself. Personal laws in Bangladesh and
Pakistan on the one
hand, and India on the other, mirror each other in their treatment of majority and minority
communities. Post-Partition changes to
Muslim family laws in Bangladesh and Pakistan benefited women
greatly, but legal benefits gained by Hindu women in India did not
spill over into the neighbouring countries.
Similarly, minority women in India do not enjoy the same benefits as their counterparts in
Bangladesh and Pakistan.
Q 1.   What are the two issues the author is discussing in the passage?
a)  Nationalism and rights of minorities

b)  The responsibility of the Supreme Court to reduce gender discrimination

c)  Uniform Civil Code (UCC) and the role of the Supreme Court versus the Executive

d)  The condition of women in Pakistan and Bangladesh

Q 2.   According to the author what should be the primary goal of UCC?

a)  National integration

b)  Uniformity of law

c)  Reducing gender discrimination

d)  Uplifting women from minority communities

Q 3.   How does the UCC result in national integration according to the Supreme Court?
a)  It established equality

b)  It removes gender discrimination

c)  It creates allegiance towards the same set of laws

d)  It creates nationalist sentiment

https://www.aspiration.link/LAW/online/QuestionPaper.jsp 1/43
31/10/2022, 15:39 Question Paper

Q 4.   Which of the following is a synonym of 'acute'?

a)  Brief

b)  Shrill

c)  Urgent

d)  Astute

Q 5.   What doesthe last paragraph of the passage imply?


a)  The condition of Muslim women in India, Pakistan, and Bangladesh will continue to worsen

b)  Irrespective of the country there is no hope for women in the Third World

c)  Women from minority communities did not reap from the benefit of changes brought about by law in their
respective countries

d)  The situation of Muslim women in Pakistan is better that those of Muslim women in India

Directions: Read the passages given below and answer the questions that follow.

Passage – 2

One may call it much ado about a candy that half the country hasn't even heard about, but the acrimony about
"revdi culture" has
only helped rev up the debate on the existential crisis looming over the cooperative federalism
envisaged by our founding fathers.
The Supreme Court should, in all its earnestness, turn its attention to the
plaque of opaqueness that has accumulated over Centre-
state relations instead of so-called freebies. It's time the
country and its opinion makers understood the Centre's stranglehold over
states on almost all fronts, which is
leading to a breakdown of trust between the two "partners".

The framers of our Constitution were aware of the apprehensions raised when India embraced a federal structure
with strong unitary
features. But they would have never dreamt of a situation where trumpeting unilateralism is
hailed as some kind of a good
governance model.

It's interesting to recall how Jawaharlal Nehru defended the unitary slant of our polity. "It would be injurious to the
interests of the
country to provide for a weak central authority which would be incapable of ensuring peace, of
coordinating vital matters of common
concern and of speaking effectively for the whole country in the international
sphere." But B R Ambedkar assured the Constituent
Assembly: "The Constitution is a federal Constitution…
The Union is not a league of states…nor are the states the agencies of the
Union, deriving powers from it. Both the
Union and the states are created by the Constitution, both derive their respective authority
from the Constitution."

I will restrict this article to fiscal federalism since "revdi culture" is the focal point of the debate although the
process of centralisation
is now all-pervasive. Over the years, the very nature of fiscal transfer has become
thoroughly centralised. For a long time, the
Planning Commission and Finance Commission were the cardinal
pillars of Centre-state relations on the fiscal front. But the ushering
in of the Goods and Services Tax (GST), the
scrapping of the Planning Commission and non-adherence to Finance Commission
recommendations have led
to an asymmetrical framework.
Q 6.   What is the synonym of 'acrimony'?

a)  animosity

b)  perspicacity

c)  felicity

d)  dexterity

Q 7.   Which statement uses the word, 'envisage',correctly?


a)  The freedom fighters of India envisaged a self-reliant nation.

b)  She envisaged the bag into the car.

https://www.aspiration.link/LAW/online/QuestionPaper.jsp 2/43
31/10/2022, 15:39 Question Paper

c)  She put the letter in the envisage.

d)  Peter envisaged the ship from the harbour.

Q 8.   What central concern does the author express in this passage?


a)  The scrapping off the Planning Commission.

b)  The views expressed by Nehru and Ambedkar.

c)  The centralising trend between the state and centre which is straining their relationship.

d)  The proliferation of 'revdi' culture.

Q 9.   What does the statement 'have led to an asymmetrical framework' mean in the context of the passage?
a)  Have led to more representation for the centre at expense of the states

b)  More free schemes are being announced by the states

c)  There is a power imbalance between the states and the centre

d)  The Planning Commission was more important that the Finance commission

Q 10.   What is the correct paraphrased sentence of B R Ambedkar's view on the centre and states?
a)  The states are extensions of the centre

b)  The states are subordinate to the centre

c)  The states and centre have their distinct responsibilities within the constitutional framework

d)  The states and centre are equal as they both are derived from the same constitutional framework

Directions: Read the passages given below and answer the questions that follow.

Passage – 3

Free speech, always a precarious ideal, has fallen on hard times. In India, it is not a value defended with passion,
enshrined firmly in
law, or insulated from being taken hostage by politics. Quite the contrary: it has become the site
at which our prejudices are
expressed, our hostilities intensified and our mutual hypocrisies exposed. It is not an
ideal around which to attempt a consensus.
Instead, it has become a pretext for warring groups to assert their
power. But the precariousness of free speech in India is also now
intimately tied to an increasingly murderous
global politics of free speech. The execution-style terrorism at the offices of Charlie
Hebdo has reignited a debate
on the threat to free expression that some brands of militant Islam pose. It has also exposed deep
fault lines in the
European discourse on free speech.

In India, the threat to freedom is serious. It is denting our reputation as a liberal democracy. This is a democracy
where artistes are
forced into exile, scholarly works are taken off the shelves, investigative journalism is in serious
peril, historical inquiry is curtailed,
political films are censored, and authors have to announce their own death.
There is a sense of foreboding about free expression.
But the roots of this foreboding run very deep.

Free Speech has always been attacked. Most societies have also placed some limits on speech- whether in the
name of regulating
the expression of hate, preventing violence, protecting individuals against libel and slander, or
outlawing obscenity. There have been
vigorous philosophical and constitutional debates about the boundaries of
such regulation. Many of these will continue. This essay
will not go into traditional legal and philosophical questions
about the boundaries of free speech. Instead it is going to ask questions
about the cultural politics of free speech.
The idea is not to resolve issues, as much as it is to provoke a deeper discussion about the
politics of speech.
Why does it feel as if the ideal of free speech is now so tied up in knots that pulling any string seems to imperil it
even more?

The attack on Charlie Hebdo was of course a classic and murderous illustration of such a trap, where every
response can be seen
https://www.aspiration.link/LAW/online/QuestionPaper.jsp 3/43
31/10/2022, 15:39 Question Paper
by the perpetrators of this attack as a vindication. It is, like most terrorist traps, by design,
a 'heads we win, tails you lose' strategy. In
a sense, calls to respect people's religion and not be provocative in the
aftermath of an attack have earned the attackers a kind of
victory; they have induced self-restraint. If, on the other
hand, such an act inflames anti-Islam feelings, as despite best efforts it
might, then also a purpose is achieved.
It produces the kind of polarisation Al-Qaeda would like. States might use the language of
bringing the killers to
justice. But in the era after 9/11, it has invariably led democratic states to commit all kinds of excesses in
different
parts of the world to the point where their moral self-confidence is dented. Again, Al-Qaeda achieves its purpose.
Q 11.   What does the writer aim to do in the passage?
a)  To provoke discussion on various aspects of free speech

b)  To condemn the supporters of free speech

c)  To oppose the supporters of free speech

d)  To convince the readers about the need for free speech

Q 12.   How does the writer describe the situation of free speech in India?
a)  Perilous

b)  Unproblematic

c)  Non-existent

d)  Useful

Q 13.   What is tone of the passage with regards to the condition of free speech all over the world?
a)  Anxious

b)  Jubilant

c)  Angry

d)  Optimistic

Q 14.   According to the passage how is free speech misused in India?


a)  It is not used at all

b)  It is used in anti-national activities

c)  It is used to express our bigotry

d)  It is used to hurt religious sentiments

Q 15.   What is the synonym of 'libel'?

a)  Rebel

b)  Liberate

c)  Defame

d)  Free

Directions: Read the passages given below and answer the questions that follow.

Passage – 4

When I worked in a second-hand bookshop - so easily pictured, if you don't work in one, as a kind of paradise
where charming old
gentlemen browse eternally among calf-bound folios - the thing that chiefly struck me was the
rarity of really bookish people. Our

https://www.aspiration.link/LAW/online/QuestionPaper.jsp 4/43
31/10/2022, 15:39 Question Paper
shop had an exceptionally interesting stock, yet I doubt whether ten per cent of
our customers knew a good book from a bad one.
First edition snobs were much commoner than lovers of
literature, but oriental students haggling over cheap textbooks were
commoner still, and vague-minded women
looking for birthday presents for their nephews were commonest of all.

Many of the people who came to us were of the kind who would be a nuisance anywhere but have special
opportunities in a
bookshop. For example, the dear old lady who 'wants a book for an invalid' (a very common
demand, that), and the other dear old
lady who read such a nice book in 1897 and wonders whether you can find
her a copy. Unfortunately she doesn't remember the title
or the author's name or what the book was about, but she
does remember that it had a red cover. But apart from these there are two
well-known types of pest by whom
every second-hand bookshop is haunted. One is the decayed person smelling of old breadcrusts
who comes
every day, sometimes several times a day, and tries to sell you worthless books. The other is the person who
orders
large quantities of books for which he has not the smallest intention of paying. In our shop we sold nothing
on credit, but we would
put books aside, or order them if necessary, for people who arranged to fetch them away
later. Scarcely half the people who ordered
books from us ever came back. It used to puzzle me at first.
What made them do it? They would come in and demand some rare and
expensive book, would make us promise
over and over again to keep it for them, and then would vanish never to return. But many of
them, of course, were
unmistakable paranoiacs. They used to talk in a grandiose manner about themselves and tell the most
ingenious
stories to explain how they had happened to come out of doors without any money - stories which, in many cases,
I am
sure they themselves believed.

We did a good deal of business in children's books, chiefly 'remainders'. Modern books for children are rather
horrible things,
especially when you see them in the mass. Personally I would sooner give a child a copy of
Petrenius Arbiter than Peter Pan, but
even Barrie seems manly and wholesome compared with some of his later
imitators. At Christmas time we spent a feverish ten days
struggling with Christmas cards and calendars, which
are tiresome things to sell but good business while the season lasts. It used to
interest me to see the brutal
cynicism with which Christian sentiment is exploited. The touts from the Christmas card firms used to
come
round with their catalogues as early as June. A phrase from one of their invoices sticks in my memory. It was:
'2 doz. Infant
Jesus with rabbits'.
Q 16.   What is the tone of the passage?
a)  Cynical

b)  Happy

c)  Nostalgic

d)  Romantic

Q 17.   In what way does the passage defy the romantic stereotypes about book shops and book-lovers?
a)  It shows that book-sellers are not interested in selling books

b)  It shows that most readers are not avid readers

c)  It shows that book-shops are meant only for students

d)  It shows that book-shops have only old books

Q 18.   What best describes the writer's attitude towards the readers who thronged his book-shop?
a)  Judgmental

b)  Loving

c)  Rude

d)  Nonchalant

Q 19.   In the context of the passage what does 'invalid' mean?

a)  Incorrect

b)  Unavailable

c)  Bed-ridden

https://www.aspiration.link/LAW/online/QuestionPaper.jsp 5/43
31/10/2022, 15:39 Question Paper

d)  Weak

Q 20.   Choose a suitable title for the passage:-

a)  Book-shop Memories

b)  The Love for Books

c)  My Happy Days at a Book-shop

d)  The Bliss of Reading

Directions: Read the passages given below and answer the questions that follow.

Passage – 5

For ages, people have debated if leaders are born or made. So too goes the debate about emotional intelligence.
Are people born
with certain levels of empathy, for example, or do they acquire empathy as a result of life's
experiences? The answer is both.
Scientific inquiry strongly suggests that there is a genetic component to emotional
intelligence. Psychological and developmental
research indicates that nurture plays a role as well. How much of
each perhaps will never be known, but research and practice
clearly demonstrate that emotional intelligence can
be learned.

One thing is certain: Emotional intelligence increases with age. There is an old-fashioned word for the phenomenon:
maturity. Yet
even with maturity, some people still need training to enhance their emotional intelligence. Unfortunately,
far too many training
programs that intend to build leadership skills-including emotional intelligence-are a waste of
time and money. The problem is simple:
They focus on the wrong part of the brain.

Emotional intelligence is born largely in the neurotransmitters of the brain's limbic system, which governs feelings,
impulses, and
drives. Research indicates that the limbic system learns best through motivation, extended practice,
and feedback. Compare this
with the kind of learning that goes on in the neocortex, which governs analytical and
technical ability. The neocortex grasps concepts
and logic. It is the part of the brain that figures out how to use a
computer or make a sales call by reading a book. Not surprisingly-
but mistakenly-it is also the part of the brain
targeted by most training programs aimed at enhancing emotional intelligence. When
such programs take, in
effect, a neocortical approach, my research with the Consortium for Research on Emotional Intelligence in
Organizations has shown they can even have a negative impact on people's job performance. Training programs
must help people
break old behavioral habits and establish new ones. That not only takes much more time than
conventional training programs, it also
requires an individualized approach.

Imagine an executive who is thought to be low on empathy by her colleagues. Part of that deficit shows itself as an
inability to listen;
she interrupts people and doesn't pay close attention to what they're saying. To fix the problem,
the executive needs to be motivated
to change, and then she needs practice and feedback from others in the
company. A colleague or coach could be tapped to let the
executive know when she has been observed failing to
listen. She would then have to replay the incident and give a better response;
that is, demonstrate her ability to
absorb what others are saying. And the executive could be directed to observe certain executives
who listen well
and to mimic their behavior.

With persistence and practice, such a process can lead to lasting results. It's important to emphasize that building
one's emotional
intelligence cannot-will not-happen without sincere desire and concerted effort. A brief seminar
won't help; nor can one buy a howto
manual. It is much harder to learn to empathize-to internalize empathy as a
natural response to people-than it is to become adept at
regression analysis. But it can be done. "Nothing great
was ever achieved without enthusiasm," wrote Ralph Waldo Emerson. If your
goal is to become a real leader,
these words can serve as a guidepost in your efforts to develop high emotional intelligence.
Q 21.   What can be said about author's attitudes towards a typical training program that intends to build leadership
skills?

a)  Instrumental in grooming leadership qualities

b)  Useless in the present times.

c)  A sheer wastage of one's resources

d)  Need to be banned

https://www.aspiration.link/LAW/online/QuestionPaper.jsp 6/43
31/10/2022, 15:39 Question Paper

Q 22.   Which of the following cannot be inferred from the passage?

a)  Meddling with the neocortex can be unfortunate.

b)  The inability to listen others indicates lack of empathy of one's colleagues.

c)  Building one's emotional intelligence cannot be completed overnight.

d)  A 75-year old woman is likely to be more emotionally intelligent than 24-year old boy.

Q 23.   What can be concluded about the limbic system as per the passage?
a)  An autonomous system that governs feelings, impulses, and drives.

b)  A system that governs feelings, impulses, and drives of a person.

c)  A part of the brain that regulates bodily responses to external temptations.

d)  An independent organ in the human skull responsible for generating impulses.

Q 24.   Which of the following correctly describes the author's tone?


a)  Contentious

b)  Aggressive

c)  Vitriolic

d)  Insightful

Q 25.   Which rhetorical device is used in the following sentence?


'These words can serve as a guidepost in your efforts to develop high emotional intelligence.'
a)  Metaphor

b)  Apostrophe

c)  Simile

d)  Synecdoche

Directions: Read the passages given below and answer the questions that follow.

Passage – 6

Classical political economists of the 18th and 19th centuries have had different views of the society. Some
economists have put
forward the thesis of a harmonious society, promoting the policies of laissez-faire. The core
belief behind this understanding is that if
everybody pursues their self-interest, then the society would operate in a
harmonious fashion. Some others have focused on the
conflicts between classes in the capitalist society, lending
economics the name of a "dismal science". While the former have been
labelled as optimists, the latter have been
commonly called pessimists. The optimists are enthusiastic about the growth prospects of
capitalism, while the
pessimists are critical about the same.

Adam Smith was a classical economist who focused on long-run economic growth with his analysis centred
around a self-interest
maximising individual. Smith's (1776) An Inquiry into the Nature and Causes of the Wealth of
Nations was concerned with increasing
the overall national income of a country. This was to be brought about by
the division of labour that would stimulate capital
accumulation, increase productivity of the workers, and increase
national output, and consequently, the wealth of nations would be
augmented.

One of Smith's popular concepts is the "invisible hand". The implication is that the self-interested actions of every
individual would
culminate in public welfare and the competitive forces in the economy would constantly work to
harmonise society as a whole. In The
Theory of Moral Sentiments, Smith (1759) states that by spending their
surplus wealth, rich people unknowingly contribute to making
https://www.aspiration.link/LAW/online/QuestionPaper.jsp 7/43
31/10/2022, 15:39 Question Paper
a distribution of the "necessaries of life" equally
among the people of the society. Similarly, in The Wealth of Nations, he asserts that
when individuals pursue their
own advantages, it collectively leads to public benefit.

Smith has often been touted as the prophet of capitalism. He wrote that the history of human beings has passed
through four major
epochs: the age of hunters, the age of shepherds, the age of agriculture, and the age of
commerce; and this history has culminated
into the age of commerce where freedom, civilisation and
self-preservation reached its highest form. He has, therefore, been labelled
as "the great optimist" on account of
his enthusiasm towards the prospects of economic growth in the commercial age, and his belief
in the rationality
of the individual. Through free trade, division of labour and capital formation, Smith further envisaged the progress
of a nation under the domain of the unrestricted spread of the market economy through laissez-faire.

However, Smith had himself discussed the issues of conflict and power structure within the economy. He had
mentioned that when
the land of a country becomes private property, the class of landlords will try to extract rent
off the land through the labour of the
workers. Further, in the chapter, "Of the Wages of Labour" in The Wealth of
Nations, Smith identified the contradiction between the
workers and the employers in the manufacturing sector.
He placed these contradictions on the difference in interest of the two
classes. While the employers are interested
in lowering the wages, the workers' interests lie in increasing the same. In this conflict,
the employers and workers
do not have the same bargaining power because of the legislative power and higher savings that the
former
possess. As a result, the employers' interests generally prevail.
Q 26.   Why has Adam Smith been called "the great optimist"?

a)  
On account of his enthusiasm towards the prospects of intellectual growth in the commercial age,
and his belief in the rationality of
the individual.
b)  Through free trade, division of labour and capital formation, Smith envisaged the progress of a nation.

c)  
He wrote that human beings have passed through four major epochs: the age of hunters, the age of
shepherds, the age of
agriculture, and the age of commerce

d)  
Adam posited that the population would outgrow natural resources and the society would ultimately
reach a situation of "misery and
vice".
 

Q 27.   As per Adam Smith, which of the following does not contribute in increasing the overall national income of a
country?
a)  Division of labour

b)  Stimulation of capital accumulation

c)  Increment in productivity of the workers

d)  Increment in national expenditure

Q 28.   As per Adam Smith, how is being rich conducive to "the invisible hand"?
a)  
Business productivity and profitability are improved when profits and losses accurately reflect what
investors and consumers want.
b)  
The constant interplay of individual pressures on market supply and demand causes the natural movement
of prices and the flow of
trade.
c)  
As wealthy individuals purpose their own interests, they distribute the necessities that all would have
received had there been an
equal division of the Earth..

d)  
Through free trade, and capital formation, a nation can progress under the domain of the unrestricted
spread of the market
economy through laissez-faire.
 

https://www.aspiration.link/LAW/online/QuestionPaper.jsp 8/43
31/10/2022, 15:39 Question Paper

Q 29.   Which of the following correctly describes the 'class antagonism' introduced by Adam Smith?
a)  When the land of a country becomes private property, the class of landlords will try to rent out the land in
exchange of labour.

b)  While the employers are interested in lowering the wages, the workers' interests lie in increasing the
same.

c)  The employers and workers do not have the same bargaining power, and the employers' interests generally
prevail.

d)  The interest of the landowners is "always opposed to the interest of every other class in the community".

Q 30.   Which of the following correctly describes the primary purpose of the author?
a)  To describe the economic outlook of Adam Smith with respect to free markets.

b)  To contrast the economic philosophy of Adam Smith with that of other economists.

c)  To highlight the upsides of the Laissez-faire economic system.

d)  To criticize the economics of optimists like 'Adam Smith'.

Passage – 1

Nepal has urged India to suspend the recruitment of Gorkhas to the Indian Army under the newly introduced
Agnipath scheme.
Foreign Minister Narayan Khadke met with India's Ambassador to Nepal Naveen Srivastava
and requested him to postpone the plan
to recruit Nepali youths under the new scheme.

Another report in the Kathmandu Post said that the meeting took place just a day before the Indian Army's scheduled
plan to recruit
Nepali youths in Butwal in Lumbini Province, for its Gorkha regiments.

Although Nepal government is positive towards facilitating recruitment of Gorkhas in the Indian Army but the
government will formally
make the decision regarding the matter after holding talks with other major political
parties as government of India has introduced the
new military recruitment, Khadke told the Indian envoy during
the meeting held at the Ministry of Foreign Affairs, the Kantipur daily
said.

While unveiling the scheme in June, the Indian government said youths between the age of 17 and-a-half and 23
years would be
inducted for a four-year tenure while 25 per cent of them will be subsequently inducted for regular
service.
Q 31.   Nepalese youths have been recruited in the Indian Army for a long time since the Tripartite agreement signed
between
Nepal, India and _____________ in 1947.
a)  Myanmar

b)  Bangladesh

c)  United Kingdom

d)  Pakistan

Q 32.   The first battalion of the Gorkha Regiment was raised as the Nasiri regiment on___________.
a)  24 April 1815

b)  24 April 1857

c)  24 April 1915

d)  24 April 1927

Q 33.   Which of the following statements is Not true regarding the benefits for Agniveers under Agnipath Scheme?
a)  
Upon the completion of the 4-years of service, a one-time 'Seva Nidhi' package of Rs. 11.71 lakhs will be
paid to the Agniveers that
will include their accrued interest thereon.

https://www.aspiration.link/LAW/online/QuestionPaper.jsp 9/43
31/10/2022, 15:39 Question Paper

b)  They will also get a Rs 48 lakh life insurance cover for the four years.

c)  In case of death, the payout will be over Rs. 4 crore, including pay for the unserved tenure.

d)  The government will help rehabilitate soldiers who leave the services after four years.

Q 34.   Which of the following statements is/are true regarding the Agnipath Scheme?
a)  Youth will be able to be recruited into the army for a short duration.

b)  It aims at providing an opportunity to the patriotic and motivated youth with the 'Josh' and 'Jazba' to join
the Armed Forces.

c)  The scheme envisions that, the average age in the forces is 32 years today, which will go down to 26 in
six to seven years.

d)  All of the above

Q 35.   Who among the following was the first and only officer of the Indian Air Force (IAF) to be promoted to
five-star rank as
Marshal of the Indian Air Force, equal to the army rank of Field Marshal?

a)  Romesh Bhandari

b)  P. Venkateswaran

c)  Markandey Singh

d)  Arjan Singh

Passage – 2

India capped off an impressive campaign at the 2022 Commonwealth Games, finishing the event in Birmingham
fourth with [1]
medals. On the final day, Shuttlers PV Sindhu and Lakshya Sen struck gold in their respective
singles' finals in badminton, while the
men's doubles pair of Satwiksairaj Rankireddy and Chirag Shetty too won
gold after winning the final. Paddler Sharath Kamal was
one of the gold medallists in table tennis, beating England's
Liam Pitchford in the men's singles final.

Weightlifters Sanket Sargar, Gururaja Poojary and Bindyarani Devi had opened India's medals tally at CWG on the
second day of
the Games. Meanwhile, Mirabai Chanu was India's first gold medallist at the CWG this edition,
winning the women's 49kg
competition.

The India men's hockey team collected silver after losing 0-7 to Australia in the final, while their women's counterpart
won bronze by
beating New Zealand in the playoff. It was another tale of 'too close, yet too far' for Harmanpreet
Kaur's India cricket team, who went
down to Australia by nine runs in the final to win silver. Boxers and wrestlers
too claimed medals, with Amit Panghal, Nikhat Zareen,
Sakshi Malik and Bajrang Punia among Indians striking
gold.
Q 36.   How many medals did India win at 2022 Commonwealth Games which has been redacted with [1] in the
passage above?
a)  36

b)  46

c)  56

d)  61

Q 37.   How many gold medals did India win at 2022 Commonwealth Games?
a)  12

b)  16

c)  22

d)  32

https://www.aspiration.link/LAW/online/QuestionPaper.jsp 10/43
31/10/2022, 15:39 Question Paper

Q 38.   What is the India rank at the 2022 Commonwealth Games?


a)  Third

b)  Fourth

c)  Sixth

d)  Tenth

Q 39.   Which of the following countries topped the medals list at the 2022 Commonwealth Games?
a)  Australia

b)  England

c)  Canada

d)  China

Q 40.   The official mascot for the Birmingham 2022 Commonwealth Games is
a)  Jerry

b)  Perry

c)  Emma

d)  Burnie

Passage – 3

Amid allegations of the Bharatiya Janata Party trying to topple the Jharkhand government, the Election Commission
on August 26
disqualified Jharkhand Chief Minister Hemant Soren as MLA over a mining case, several media
outlets reported.

The development came just hours after the Jharkhand CM presided over a meeting of the legislators and ministers
of the ruling
Jharkhand Mukti Morcha-led alliance at his residence in Ranchi to discuss the political crisis that has
gripped the state

The ECI had made the recommendation to Governor Ramesh Bais after he sought its advice on allegations
levelled against Soren
by the BJP and the latter seeking action against him for holding a mining licence which is
equivalent to holding an "office of profit".

Earlier this year, the saffron party had written to the Governor seeking the Jharkhand CM's disqualification as MLA
under Section
9(A) of the Representation of People's Act. They had alleged that he misused his position to obtain
a stone mine's licence. Soren,
however, had applied for cancellation of the lease in the same month (February)
itself. He claimed that the mining lease did not fall
under the purview of the office of profit rules as he got the
10-year lease in 2008 when he was not the chief minister.
Q 41.   Recently, Election Commission (EC) recommended the disqualification of Jharkhand Chief Minister Hemant
Soren as MLA
under the________________.
a)  Representation of the People Act, 1949

b)  Representation of the People Act, 1951

c)  Representation of the People Act, 1954

d)  Representation of the People Act, 1956

Q 42.   Which of the following statements is/are true regarding the disqualification under the Constitution of India?
a)  
If he holds any office of profit under the Union or state government (except that of a minister or any other
office exempted by state
legislature).

https://www.aspiration.link/LAW/online/QuestionPaper.jsp 11/43
31/10/2022, 15:39 Question Paper

b)  If he is of unsound mind and stands so declared by a court.

c)  
If he is not a citizen of India or has voluntarily acquired the citizenship of a foreign state or is under any
acknowledgement of
allegiance to a foreign state.
d)  All of the above

Q 43.   The Tenth Schedule of the Indian Constitution lays down the grounds and process by which legislators may
be disqualified
on grounds of defection by the Presiding Officer of a legislature based on a petition by any
other member of the House. The Tenth
Schedule of the Indian Constitution was inserted by the_____________.
a)  32nd Amendment Act

b)  42nd Amendment Act

c)  52nd Amendment Act

d)  62nd Amendment Act

Q 44.   Under the Anti-defection Law, a nominated member of a House becomes disqualified for being a member of
the House if he
joins any political party after the expiry of ___________months from the date on which he
takes his seat in the House.

a)  Three

b)  Six

c)  Nine

d)  Ten

Q 45.   Under Article ___________ and Article ___________of the Constitution, an MP or an MLA (or an MLC) is
barred from
holding any office of profit under the Central or State government.
a)  Article 100 (1) and Article 190(1) of the Constitution

b)  Article 101 (1) and Article 189(1) of the Constitution

c)  Article 102 (1) and Article 191(1) of the Constitution

d)  Article 103 (1) and Article 193(1) of the Constitution

Passage – 4

India is projected to surpass [1] as the world's most populous country next year, according to a report by the United
Nations which
said that the world population is forecast to reach eight billion by mid-November 2022.

The World Population Prospects 2022 by the United Nations Department of Economic and Social Affairs, Population
Division, said
that the global population is projected to reach eight billion on November 15, 2022.

The global population is growing at its slowest rate since 1950, having fallen under 1% in 2020. The latest projections
by the United
Nations suggest that the world's population could grow to around 8.5 billion in 2030 and 9.7 billion in
2050. It is projected to reach a
peak of around 10.4 billion people during the 2080s and to remain at that level until
2100.

"This year's World Population Day falls during a milestone year, when we anticipate the birth of the Earth's eight
billionth inhabitant.
This is an occasion to celebrate our diversity, recognise our common humanity, and marvel at
advancements in health that have
extended lifespans and dramatically reduced maternal and child mortality rates,"
U.N. Secretary-General António Guterres said.

"At the same time, it is a reminder of our shared responsibility to care for our planet and a moment to reflect on
where we still fall

https://www.aspiration.link/LAW/online/QuestionPaper.jsp 12/43
31/10/2022, 15:39 Question Paper
short of our commitments to one another," he added. The report said that "India is projected to
surpass [1] as the world's most
populous country during 2023."
Q 46.   According to a report by the United Nations, India is projected to surpass [1] as the world's most populous
country next year,
which of the following countries has been redacted with [1] in the passage above?
a)  USA

b)  Russia

c)  China

d)  United Kingdom

Q 47.   The World Population Day is observed every year on __________.


a)  April 11

b)  May 11

c)  June 11

d)  July 11

Q 48.   The theme for the World Population Day 2022 is______________.
a)  The impact of the Covid-19 pandemic on fertility.

b)  How to safeguard the health and rights of women and girls now.

c)  A world of 8 billion: Towards a resilient future for all - Harnessing opportunities and ensuring rights and
choices for all.

d)  Family planning is a human right.

Q 49.   Which is the only state in the country to record a negative growth (–0.47%) in decadal population growth in
the 2011
census?
a)  Goa

b)  Sikkim

c)  Manipur

d)  Nagaland

Q 50.   The 2021 Census of India also the ___________ Indian Census, is intended to be carried out in 2023.
a)  10th

b)  12th

c)  16th

d)  20th

Passage – 5

India's rank improved one place to [1] among 180 countries in a corruption perception index (CPI) of 2021, according
to a new report
by Transparency International which, however, raised concern over the country's democratic
status.

The index, which ranks 180 countries and territories by their perceived levels of public sector corruption according
to experts and
business people, uses a scale of 0 to 100, where 0 is highly corrupt and 100 is very clean.
[2], Finland, New Zealand and Norway
topped the list with the highest score. [3], Syria and Somalia remain at the
bottom of the index.

https://www.aspiration.link/LAW/online/QuestionPaper.jsp 13/43
31/10/2022, 15:39 Question Paper

Among those with weak scores are some of the world's most populous countries such as China (45) and India
(40), and other large
economies such as Indonesia (38), Pakistan (28) and Bangladesh (26), the report by the
anti-corruption watchdog said.

India's rank improved by one place to [1] in 2021 from 86th in 2020, according to the index. Except Bhutan, all of
India's neighbours
are ranked below it. Pakistan dropped 16 spots in the index and was ranked at 140.

Calling the case of India particularly worrying, the report released said while the country's score has remained
stagnant over the past
decade, some of the mechanisms that could help reign in corruption are weakening.

There are concerns over the country's democratic status, as fundamental freedoms and institutional checks and
balances decay.
"Journalists and activists are particularly at risk and have been victims of attacks by the police,
political militants, criminal gangs and
corrupt local officials, the report based on the index said.
Q 51.   What is the India rank in the corruption perception index (CPI) 2021 released by the Transparency International
which has
been redacted with [1] in the passage above?
a)  80

b)  85

c)  90

d)  96

Q 52.   Which of the following countries is the least corrupt country in the corruption perception index (CPI) 2021
released by the
Transparency International which has been redacted with [2] in the passage above?
a)  Poland

b)  Canada

c)  Denmark

d)  Switzerland

Q 53.   Which of the following countries is the most corrupt country in the corruption perception index (CPI) 2021
released by the
Transparency International which has been redacted with [3] in the passage above?
a)  South Sudan

b)  Afghanistan

c)  Sri Lanka

d)  Somalia

Q 54.   Transparency International is a nonprofit and non-governmental organization based in____________.


a)  Tokyo

b)  Berlin

c)  Paris

d)  New York

Q 55.   India Against Corruption (IAC) is an anti-corruption movement in India which was particularly prominent
during the anti-
corruption protests of 2011 and 2012, concerned with the introduction of the Jan Lokpal bill.
Who among the following is/are
associated with India Against Corruption (IAC)?
a)  Arvind Kejriwal

https://www.aspiration.link/LAW/online/QuestionPaper.jsp 14/43
31/10/2022, 15:39 Question Paper

b)  Anna Hazare

c)  Ramachandra Guha

d)  Both (a) and (b)

Passage – 6

The Shanghai Cooperation Organisation is all set for expansion with [1] joining the grouping as its ninth member
while Belarus has
applied for membership, its Secretary General Zhang Ming said.

The decision to admit [1] was made in last year's Dushanbe summit and Belarus has submitted its application,
Zhang told a media
briefing here ahead of the grouping's summit to be held in Samarkand in Uzbekistan on
September 15-16.

He said [1] will become a full member after completing the formalities while Shanghai Cooperation Organisation
(SCO) member
states who follow the system of consensus in deciding admission of new members will take a call
on Belarus's application.

This is the first expansion of the SCO after India and [2] were admitted to the grouping in 2017. The SCO is an
eight-member
economic and security bloc consisting of China, Russia, Kazakhstan, Kyrgyzstan, Tajikistan,
Uzbekistan, India and [2].

When asked whether Chinese President Xi Jinping will attend the Samarkand summit, Zhang said so far, all
participating countries
have confirmed the attendance of their leaders but the format of attendance is not finalised.
The SCO summits in the last two years
were held in the virtual format due to COVID-19.
Q 56.   The Shanghai Cooperation Organisation is all set for expansion with [1] joining the grouping as its ninth
member, which of
the following countries has been redacted with [1] in the passage above?
a)  Iraq

b)  Iran

c)  Israel

d)  Saudi Arabia

Q 57.   In 2017, India and [2] were admitted to the SCO grouping, which of the following countries has been redacted
with [2] in the
passage above?
a)  Latvia

b)  Pakistan

c)  Vietnam

d)  Indonesia

Q 58.   The Shanghai Cooperation Organisation (SCO) is a Eurasian political, economic and security organization.
The SCO
Charter was signed on 7 July 2002 and entered into force on__________.
a)  19 August 2002

b)  19 September 2003

c)  19 July 2004

d)  19 July 2005

Q 59.   Which of the following statements is /are true regarding the Shanghai Cooperation Organisation (SCO)?

a)  The supreme decision-making body in the SCO is the Heads of State Council (HSC).

https://www.aspiration.link/LAW/online/QuestionPaper.jsp 15/43
31/10/2022, 15:39 Question Paper

b)  
The Heads of State Council (HSC) meets once in a year and adopts guidelines and decisions on all
important matters of the SCO.

c)  
The SCO has two permanent bodies; the first is the Executive Committee of the Regional Anti-Terrorist
Structure (RATS) based in
Tashkent and second is the SCO Secretariat based in Beijing.
d)  All of the above

Q 60.   In September 2021, the 6th Edition of SCO exercise ‘Peaceful Mission’ is hosted by____________.
a)  India

b)  China

c)  Russia

d)  Kazakhstan

Passage – 7

India's national emblem cast on the roof of the new Parliament building was unveiled by PM Narendra Modi.
Several Opposition
parties have slammed the expression of the four lions of the national emblem in the sculpture,
and alleged that Modi government has
violated the Constitution. However, Central government justified the national
emblem's cast saying it was done after research and a
well-laid procedure.

The national emblem is one of the most visible symbols of national identity and is used as the seal of the Republic.
Ours was
adopted from the Lion Capital of one of the Ashoka Pillars on [1]. The symbol was adopted along with the
motto 'Satyamev Jayate',
taken from the [2] and meaning 'truth always wins'. Our national emblem has four lions
mounted back to back on a circular abacus,
facing four different directions. They represent courage, pride, power
and confidence.

A circular abacus on which the lions stand is adorned with the engravings of a bull, a horse, and an elephant.
Along with the lions, it
is believed that the animals represent the four stages of Gautam Buddha's life.
Lions indicate the stage of achieving enlightenment.

The bull symbolises Taurus, the zodiac sign of Buddha. The elephant denotes his outset. The horse symbolises
his ride after leaving
the citadel where he gave his first sermon. The animals are separated by Ashok Chakras or
Dharmachakras. The above-mentioned
structure rests on an inverted lotus, which was chosen as the National
Flower of India. However, it is not a part of the Emblem.
Q 61.   National Emblem of India is a representative seal of the Republic of India that is adapted from the Lion
Capital of Ashoka
Pillar. India adopted it as the State Emblem on [1], which of the following has been redacted
with [1] in the passage above?
a)  January 26, 1949

b)  January 26, 1950

c)  January 26, 1951

d)  January 26, 1952

Q 62.   National Emblem is the symbol of authority and is present in all the official communications of the government.
The symbol
was adopted along with the motto 'Satyamev Jayate', taken from the [2] and meaning 'truth
always wins'. Which of the following has
been redacted with [2] in the passage above?
a)  Aitareya Upanishad

b)  Mundaka Upanishad

c)  Taittiriya Upanishad

d)  Prasna Upanishad

https://www.aspiration.link/LAW/online/QuestionPaper.jsp 16/43
31/10/2022, 15:39 Question Paper

Q 63.   The Lion Capital of Ashoka is the capital, or head, of a column erected by the Mauryan emperor Ashoka in
____________.
a)  Sarnath

b)  Bodh Gaya

c)  Pataliputra

d)  Vaishali

Q 64.   How many animals are there in the national emblem of India?
a)  3

b)  4

c)  5

d)  6

Q 65.   What is the name of the ongoing redevelopment plan to revamp the Central Vista which is the central
administrative area of
India located near Raisina hill in New Delhi?
a)  Central Cabinet Vista Redevelopment Project

b)  Central Vista Redevelopment Project

c)  Delhi Central Redevelopment Project

d)  Delhi Raisina Vista Redevelopment Project

Directions for questions 66 to 105: You have been given some passages followed by questions based on each
passage. You are
required to choose the most appropriate option which follows from the passage.
Only the information given in the passage should be
used for choosing the answer and no external knowledge of
law howsoever prominent is to be applied.

Passage – 1

International law of armed conflict has always distinguished between civilians and military. Two days after Putin
announced "special
military operations", media showed that several went to defend their home. Soon after, Ukraine's
defence ministry urged citizens to
make Molotov cocktails, and the National Guard of Ukraine tweeted an infographic
on how to make it. Ukraine brought its case
before the International Court of Justice under the Genocide Convention
of 1948 and has requested an injunction, allowing Ukraine
to have a prompt hearing. Now that the ICJ is soon going
to hear Ukraine's arguments against Putin's invasion, what will be the
position of these Ukrainian citizens who
have taken up arms against the onslaught of the Russian Army?

Before the big legal fight in the ICJ, one should know of the protections that Geneva Convention III offers for
combatants and
civilians. Article 4A of the Convention has several categories of persons who take part in hostilities.
If ever caught by the enemy
forces, combatants are then protected as 'prisoners of war'. If a mercenary is ever
caught by the 'other side', they may be granted
'prisoner of war' status, but it depends on the State. Article 4A (1)
- grants combatant status to members of the armed forces, etc and
even members of State law enforcement who
have been formally incorporated into the armed forces of their State. Article 4A (2)
distinguishes between informal
volunteer groups who are then not entitled to either 'combatant' status nor 'prisoner of war' privileges.
Under Article
4A (2), combatants should wear a distinct sign and should openly carry arms.

'Lawful combatants' enjoy 'combatant immunity', which gives right to participate in hostilities and to enjoy immunity
from their warlike
acts, which also means they can be lawfully targeted by the enemy at any time.

Levée en masse is a French term used for a policy of mass national conscription, often in the face of invasion.
And under Article 4A
(6), civilians who take part in a levée en masse have a combatant status. While combatants
under this category do not need a
command structure, they still need to openly carry arms and abide by all laws of
armed conflict.

Civilians are defined in Additional protocol I, as non-combatants. Civilians enjoy all kinds of immunity and attacking
them is a war
crime. As stated by Dustin Lewis "a war crime is any conduct - whether an act or an omission - that
fulfils two cumulative criteria,
https://www.aspiration.link/LAW/online/QuestionPaper.jsp 17/43
31/10/2022, 15:39 Question Paper
first, the conduct must be committed with a sufficient connection to an armed
conflict. Second, the conduct must constitute a serious
violation of the law that has been criminalised by international
treaty or customary law".

While 'direct participation in hostilities' of civilians is a debated topic, it is generally accepted that a civilian loses
their immunity once
they directly participate in hostilities. Therefore, all persons directly taking part in hostilities that
do not fall under any of the above-
mentioned categories are not combatants, nor do they get the prisoner of war or
civilian protections.
Q 66.   Which of the following is the primary legal question that has been dealt by the author in the passage?
a)  
The author has tried to find out the position of Russia in front of International court of justice and its
argumentation defending their
acts
b)  The author analyses how Ukraine is not manifestly wrong in asking their citizens to take up arms against
the Russian tyrants.

c)  
The author analyses that what will be the position of these Ukrainian citizens who have taken up arms
against the onslaught of
Russian Army
d)  The author discusses how the rule of law is being violated by the act of Russian invasion of Ukraine

Q 67.   Which of the following statements is true in the light of the above given passage?

a)  
Ukraine brought its case before the International Court of Justice under the Nuclear power nations
non-aggression act of 1948 and
has requested an injunction.
b)  
'Lawful combatants' do not enjoy 'combatant immunity', which is the right to participate in hostilities and
the same is given only to
the registered military.
c)  Levée en masse is a French term used for a policy of mass national conscription, often in the face of
invasion

d)  None of the above

Q 68.   Russia and Ukraine are involved in a military combat. One Ukrainian soldier A is captured by the Russian
soldiers during
the war. Does A receive any protection under legislative provisions after being caught?

a)  A is a prisoner and does not have any protection. He is at the mercy of the Russian army and Lord Putin.

b)  A is a 'prisoner of war' and is protected under the relevant provisions of Geneva convention

c)  The Geneva convention is not applicable to the Russia-Ukraine conflict and therefore A has no protection.

d)  A receives protection for being the ally of the west and he should not be treated inhumanely.

Q 69.   B was a Ukrainian citizen who was helping the country in the war. He used a knife as his weapon of war and
wore a Gucci -
T shirt, Track pants and a Bikers shoe. B was caught by the Russian soldiers. Can B be
considered as a Ukrainian soldier in the
given situation?
a)  B can be considered as a Ukrainian soldier as he is fighting for the cause of Ukraine.

b)  B cannot be considered as a Ukrainian soldier as he is not wearing the requisite uniform and not carrying
any explicit weapon.

c)  Weapon and outfits do not make one a soldier, courage alone does and therefore B is a Ukrainian soldier
by heart

d)  The passage hasn't provided enough information to arrive at the correct answer.

Q 70.   Which of the following statements is false in the light of the above given passage?
a)  
While 'direct participation in hostilities' of civilians is a debated topic, it is generally not accepted that a
civilian loses their immunity
once they directly participate in hostilities.

https://www.aspiration.link/LAW/online/QuestionPaper.jsp 18/43
31/10/2022, 15:39 Question Paper

b)  Civilians on the other hand, are defined in Additional protocol I, as all persons who are non-combatants

c)  Article 4A of the Convention has several categories of persons who take part in hostilities.

d)  
Article 4A (2) distinguishes between informal volunteer groups who are then not entitled to either 'combatant'
status nor 'prisoner of
war' privileges
 

Directions for questions 66 to 105: You have been given some passages followed by questions based on each
passage. You are
required to choose the most appropriate option which follows from the passage.
Only the information given in the passage should be
used for choosing the answer and no external knowledge of
law howsoever prominent is to be applied.

Passage – 2

The Surrogacy (Regulation) Bill (SRB) 2021 is the successor to the Surrogacy (Regulation) Bill 2016, which could
not be passed by
the Parliament of India. The Bill defines surrogacy as a practice whereby one woman bears and
gives birth to a child for an intending
couple with the intention of handing over such child to the intending couple
after the birth. It seeks to prohibit commercial surrogacy,
which includes trading human embryos and gametes
and buying and selling of the services of a surrogate by a monetary reward
except for insurance but allows
altruistic surrogacy, which involves no monetary incentives or rewards to the surrogate mother other
than the
medical expenses incurred and insurance coverage.

SRB 2021 envisages that surrogacy-related medical procedures shall take place only at designated surrogacy
clinics. It also
regulates them through registration and prohibits taking services of unqualified professionals or
inducing women to become
surrogates through advertisements or other means.

The intending couple would also need a 'certificate of essentiality' and a 'certificate of eligibility issued by the
appropriate authority in
order to be eligible to opt for the procedure. A certificate of essentiality will be issued only if
infertility affects the intending couple and
certain other conditions.

Ban on Surrogacy

Judicial View
The Supreme Court's decision in Indian Hotels and Restaurants Association and Others v State of Maharashtra in
the context of bar
dancers in Maharashtra would be suitable to evaluate on the subject of what kind of labour
women may and cannot undertake. The
Supreme Court reaffirmed an earlier decision holding that a ban on a
woman's right to work cannot be imposed solely on the basis of
moral disapproval if there is no objective evidence
that the activity harms women. The Court had previously struck down a total ban
on women dancing in bars, and
in this case, the Maharashtra legislature's attempt to "regulate" it ran afoul of constitutional
guarantees of equality,
freedom of expression

Similarly, after the Supreme Court's decision in Justice K S Puttaswamy (Retd) v Union of India, the conceptions
of right to privacy
and freedom to choose have extended considerably. While the Court said that the right to privacy
is a basic right, it also stated that it
has three parts, the most important of which is "decisional autonomy." Decisional
autonomy refers to the freedom to make personal
decisions about having children, marriage, clothing, food, beliefs,
and so on. While the decision has significant flaws, any regulation
that aims to restrict women's ability to manage
their own bodies must pass the three-part standard given forth by the Supreme Court
to show why the limitation is
permissible under the Constitution. While the 2021 law would easily satisfy the "legality" test, it would
still need to
demonstrate that its provisions are necessary and appropriate under the criteria of the judgement's test.
Q 71.   What does the Surrogacy (Regulation) Bill (SRB) 2021 envisage?
a)  The surrogacy (regulation) bill 2021 envisages to remove the age old practices and replace them with
modern ideas.

b)  
The surrogacy (regulation) bill 2021 envisages that surrogacy-related medical procedures shall take
place only at designated
surrogacy clinics.
c)  
The surrogacy (regulation) bill 2021 envisages that surrogacy-related medical procedures shall take
place only under the watchful
eyes of government hospitals.
d)  The surrogacy (regulation) bill 2021 envisages to develop well structured surrogacy clinics across each
district in India.

 
https://www.aspiration.link/LAW/online/QuestionPaper.jsp 19/43
31/10/2022, 15:39 Question Paper

Q 72.   B and D were happily married. B was a medical researcher and he believed a child born through surrogacy
would develop
more immunity and would be healthier. So, B and D decide to have a baby through surrogacy.
Decide.
a)  B and D are permitted to have a surrogate baby as it is their decision to have a baby according to their
choice.

b)  B and D cannot have a surrogate baby and the same is prohibited

c)  B and D can have a surrogate baby after getting the 'certificate of essentiality'.

d)  B and D shall have to write an appeal to the public authorities to allow them to have a surrogate baby.

Q 73.   Which of the following statements is false in the light of the above given passage?
a)  Now the ambit of right to privacy has been extended to a broad ambit.

b)  
Surrogacy can be defined as a method where one woman bears and gives birth to a child for an intending
couple with the intention
of handing over such child to the intending couple after the birth.
c)  
The intending couple would also need a 'certificate of essentiality' and a 'certificate of eligibility issued by
the appropriate authority.
d)  
The Supreme Court reaffirmed an earlier decision holding that a ban on a woman's right to work can be
imposed solely on the
basis of moral disapproval.
 

Q 74.   Which one of the following is the reasoning behind the supreme court striking down a total ban on women
dancing in bars?

a)  Such a provision is violative of article 21 of the Indian constitution.

b)  Such a provision is violative of constitutional guarantees of equality, freedom of expression

c)  Such a provision is arbitrary and violative of rule of law.

d)  None of the above

Q 75.   What does 'decisional autonomy' mean in context of the above given passage?
a)  
Decisional autonomy refers to the freedom to make personal decisions about having children, marriage,
clothing, food, beliefs, and
so on.
b)  Decisional autonomy means the discretion of the court to take up certain autonomy in order to uphold the
ends of justice

c)  
Decisional autonomy means the ability of the court to intervene with the legislative overpowering and
ensure that justice is reached

d)  The passage does not explain the said concept.

Directions for questions 66 to 105: You have been given some passages followed by questions based on each
passage. You are
required to choose the most appropriate option which follows from the passage.
Only the information given in the passage should be
used for choosing the answer and no external knowledge of
law howsoever prominent is to be applied.

Passage – 3

It has been observed that the ambiguous text of provisions causes abuse of investigation powers by police officers
often leading to
harsh ramifications for ostensible innocent parties and accused persons. One of such provisions
is Clause (1) of Section 102 of
CrPC wherein police officers are empowered to freeze the bank accounts of a
person. However, the issue that surrounds the
interpretation of Section 102(1) is the condition precedent for
seizing the bank accounts of a person. The conundrum is that whether
this condition precedent before seizing the
bank account is that the transactions of the bank account must create the suspicion of the
commission of any
offence by the bank account holder, or simply that the bank account holder must be an accused in the

https://www.aspiration.link/LAW/online/QuestionPaper.jsp 20/43
31/10/2022, 15:39 Question Paper
commission
of any offence and there should be a nexus between the bank account and the alleged offence.

Clause (1) of Section 102 states that "Any police officer may seize any property which may be alleged or suspected
to have been
stolen, or which may be found under circumstances which create suspicion of the commission of
any offence." The term "create" has
created an ambiguity that whether the discovery of the offence should be a
sequel to the discovery of suspicious bank transactions
and not the other way around. On this aspect, the Supreme
Court has taken a view in the case of Tapas D.N. Neogy that there is no
such requirement that the bank transactions
must create the suspicion of an offence in every case. It was held that the police officer
can seize the bank
account if there is any direct link with the commission of the offence for which the police officer is investigating
into.
However, subsequently, High Court of Madras in R. Chandrasekar vs. Inspector of Police held that the discovery of
the offence
must precede the discovery of the suspicious transactions. The Court concluded that "There are no
circumstances attendant upon
the bank account or its operation that have led the police to suspect that some
offence has been committed somewhere.

The allegation of the prosecution is that the bank account in this case is a sequel to the discovery of the commission
of the offence.
This is not sufficient to attract Section 102 of Cr. P.C. as it cannot be since that the bank account
has been traced or discovered in
circumstance which have made the police aware of the commission of an
offence." Then, the High Court of Allahabad in Suninder
Sandha considered the decision of Apex Court in Tapas
D.N. Neogy and held that discovery of any offence can precede the
discovery of suspicious bank transactions.
It was held that "Thus, various judgments of the High Courts cited by learned counsel for
the applicant on the
proposition that powers under Section 102 of the Code, are to be exercised only when discovery of a property
leads to suspicion of a commission of an offence, does not represent the correct statement of law in view of
Neogy.
Q 76.   D was a beggar in Hastinapur. He used to beg around the streets of Rajghat and fend for food. Once B,
one police in
Hastinapur found a golden statute of Lord Vishnu with D. B immediately seized the same.
D now approaches the court of law
stating the act of B to violative of his fundamental rights. Is the act of B
is justified?
a)  The act of B is not justified as he does not have the power to seize valuable properties just because he
is a public officer.

b)  The act of B is not justified as there is no legal duty on B to capture an authority from the hands of D.

c)  The act of B is justified under the relevant provisions of code of criminal procedure code 1973.

d)  The act of B is only justified if D can establish the reasoning behind his action of seizing the property from
D.

Q 77.   Which of the following statement is false in the light of the above given passage?
a)  The passage suggests that ambiguous text of provisions causes abuse and misuse of investigation.

b)  
The allegation of the prosecution in the case of R. Chandrasekar is that the bank account in this case is
a sequel to the discovery
of the commission of the offence.

c)  
The powers under Section 102 of the Code, are to be exercised only when discovery of a property leads
to suspicion of a
commission of an offence, does not represent the correct statement of law in view of
Neogy
d)  
The police officer cannot seize the bank account if there is a direct link to a case with the commission of
the offence for which the
police officer is investigating into.
 

Q 78.   Why was the allegation of the prosecution not enough to the freeze the bank account in the case of R.
Chandrasekar?

a)  The allegation of the prosecution was not enough as the freezing of bank account only happens in
exceptional circumstances.

b)  
It cannot be since that the bank account has been traced or discovered in circumstance which have
made the police aware of the
commission of an offence
c)  
The allegation did not have enough evidence to prove that the prosecution not enough to the freeze the
bank account in the case
of R. Chandrasekar
d)  The passage fails to provide analysis for why the allegation of the prosecution not enough to the freeze
the bank account.

https://www.aspiration.link/LAW/online/QuestionPaper.jsp 21/43
31/10/2022, 15:39 Question Paper

Q 79.   What did the High Court of Allahabad in Suninder Sandha consider?
a)  The HC considered a judgement of the supreme court and took its help to arrive at the decision.

b)  The HC considered discovery of any offence can precede the discovery of suspicious bank transactions.

c)  The HC considered the application of section 102(1) in the case of Suninder Sandhu.

d)  Both a and b are correct answer.

Q 80.   How has the section 102(1) of the CrPC created ambiguity regarding the freezing of bank accounts?

a)  
The term "create" has created an ambiguity that whether the discovery of the offence should be a sequel
to the discovery of
suspicious bank transactions and not the other way around.

b)  
The term 'freeze' under the section 102(1) has created misunderstanding and confusion among the
legal luminaries bringing CrPC
under hammer.

c)  The application of section 102(1) has been varied and has led to confusion among the professionals.

d)  None of the above

Directions for questions 66 to 105: You have been given some passages followed by questions based on each
passage. You are
required to choose the most appropriate option which follows from the passage.
Only the information given in the passage should be
used for choosing the answer and no external knowledge of
law howsoever prominent is to be applied.

Passage – 4

The phenomenon of disruptive passengers is not new, yet, the number of incidents is constantly on the rise.
The International Air
Transport Association released data in 2017 which stated that there was 1 unruly passenger
incident per 1,053 flights. In 2017 alone,
approximately 8,371 incidents of unruly passenger behaviour were reported.
The definition of a disruptive passenger is laid down in
Annex 17 of the International Civil Aviation Organisation
(ICAO) Chicago Convention. This Convention on International Civil Aviation
Security Safeguarding International
Civil Aviation Against Acts of Unlawful Interference defines an "unruly passenger" as a
passenger, who does not
respect the rules of conduct at the airport or on-board a flight or fails to follow the instructions issued by the
airport
staff or crew members. Owing to this behaviour, such a passenger disturbs the good order and discipline at an
airport or on-
board an aircraft. A similar definition has been adopted by the Indian law.

A disruptive passenger is someone who by their actions or stated intentions jeopardizes or might jeopardize the
safety of the aircraft,
persons aboard it, or property therein. The International Air Transport Association (IATA) has
promulgated a 'non-exhaustive' list of
behaviour which counts as unruly or disruptive. The list mentions the following
acts:
1. Illegally consuming narcotics,
2. Smoking cigarettes in the aircraft,
3. Consuming excessive alcohol,
4. Refusing to comply with safety instructions,
5. Verbal or physical confrontation with crew members or other passengers,
6. Sexual harassment/abuse,
7. Making threats towards the crew or other passengers, and
8. Other types of reckless behaviour, including screaming, banging head on the seatbacks, etc.

International Air Transport Association is a private organisation consisting of members from nearly 120 countries.
The aim of this
organisation is working for the promotion of the interest of various airlines. Pursuant to this, IATA
has devised several guidelines and
recommended practices that have over time become the standard operating
procedure for airlines dealing with unruly passengers.
IATA has recommended that every member should develop
a company policy to deal with disruptive passengers. Further, it has also
formulated a basic guide to deal with
unruly passengers. The IATA Guidance on Unruly Passenger Prevention and Management,
2015 provides numerous
ways on how airlines can deal with unruly passengers. This guide also recommends airlines to enforce
their
respective alcohol policy so that troublemaking passengers are identified at the outset. The first-ever attempt at
framing
international legislation for governing offences and certain other acts committed on board an aircraft was
the Tokyo Convention,
1963. This legal framework has been ratified by 186 countries. Article 1 of the Convention
talks about the applicability of this law.
https://www.aspiration.link/LAW/online/QuestionPaper.jsp 22/43
31/10/2022, 15:39 Question Paper
The provisions of this Convention are applicable in cases wherein the
conduct complained of is either:
1. An offence under any applicable penal law or criminal code; or
2. It is an act which falls short of criminal conduct but is capable of jeopardizing the safety of the aircraft or the
passengers. It could
also affect the general good order and discipline as well as the overall safety.

Therefore, according to the Tokyo Convention, it is not essential that such disruptive behaviour meets the threshold
of criminal
conduct. The mere possibility of an act which is capable of jeopardising safety and security is sufficient.
Q 81.   Racheal was leaving for Paris. She was seated in the flight and was waiting for the flight to take off.
Her friend Phoebe
called her to tell that there was a problem in the left philange of the aeroplane.
Racheal had a panic attack and started acting
obnoxiously, this led to a panic situation in the entire flight.
Flight attendants had tried their best to convince Racheal that there did
not exist a part called Philange,
but in vain. This mishap delayed the flight for 2 hours. The flight management wanted to sue
Racheal for her
act, but she had not committed any act that could have been classified as a crime under countries penal
codes.
The said country had ratified the Tokyo convention. Can a suit be filed against Racheal?
a)  A suit cannot be filed against Racheal as she has not committed any act that is violative of any penal
laws.

b)  
A suit can be filed against Racheal according to article 1(2) of the Tokyo convention which allows the
prosecution of an individual
whose action affects the general safety and discipline of the plane.

c)  
A suit can be filed against Racheal in accordance with the IATA convention which allows the prosecution
of an individual whose
action affects the general safety and discipline of the plane.

d)  No suit can be filed against Racheal as her panic was reasonable. Phoebe foresees events.

Q 82.   Racheal was leaving for Paris. The flight was already set in motion for take-off. Suddenly she realized that
Ross was the
love of her life and could not live without him. She started yelling and acting violently. She was
yelling at the pilots to turn the
aeroplane back to the airport. As her behaviour went out of control the pilots
had to give in and turn the aeroplane back to the
airport. Can she be considered as an 'unruly passenger'
under Article 17 of International Civil Aviation Organisation (ICAO)
Chicago Convention?
a)  No, she cannot be considered as an unruly passenger, she was doing it for the love of her life.

b)  True love stands above 10 minutes delay, Racheal should be pardoned.

c)  
Racheal can be considered as an unruly passenger under article 17 of the International Civil Aviation
Organisation (ICAO) Chicago
Convention.
d)  Ross has to be charged, as it is his mistake that he confessed his love while she was boarding the
aeroplane.

Q 83.   Racheal had taken a flight to Paris. When the flight was about to fly, she realised that Ross was her true love
of life. This
severely affected her and traumatised her leading to a mild panic attack which made her
unconscious. The flight returned to the
airport and Racheal was taken to the hospital. Can Racheal be
charged for disruption of discipline of the flight?
a)  No, But Ross has to be charged as it is because of him Racheal had a panic attack.

b)  No Racheal had a health upset which was a natural cause and cannot be charged for the same.

c)  Yes, Racheal should not have thought about Ross in a plane, Ross is unpredictable.

d)  Yes, thinking about Ross in an aeroplane is a negligent act by Racheal.

Q 84.   Which of the following statements is false in the light of the given passage?
a)  
A disruptive passenger is someone who by their actions or stated intentions jeopardizes or might
jeopardize the safety of the
aircraft, persons aboard it, or property therein.
b)  International Air Transport Association is a public organisation consisting of members from nearly 125
countries.

https://www.aspiration.link/LAW/online/QuestionPaper.jsp 23/43
31/10/2022, 15:39 Question Paper

c)  Article 1 of the Tokyo convention discusses the applicability of the laws.

d)  
The IATA Guidance on Unruly Passenger Prevention and Management, 2015 provides numerous ways
on how airlines can deal
with unruly passengers.
 

Q 85.   Which of the following is not an association/law that deals with safety of aeroplanes and its movements?

a)  International Air Transport Association

b)  Tokyo convention 1963

c)  International Civil Aviation Organisation

d)  Aviation organisation of southeast Asian nations

Directions for questions 66 to 105: You have been given some passages followed by questions based on each
passage. You are
required to choose the most appropriate option which follows from the passage.
Only the information given in the passage should be
used for choosing the answer and no external knowledge of
law howsoever prominent is to be applied.

Passage – 5

A person is said to 'instigate' another to an act, when he actively suggests or stimulates him to the act by any
means of language.
'Instigate' literally means to goad or urge forward or to provoke. A person instigates with
intention to incite or encourage the doing of
an act by another. Instigation involves a physical act or omission as
well as a mental act. A mere acquiescence or permission does
not amount to instigation. Instigation necessarily
connotes some active suggestion, support or stimulation to the commission of the
act itself. It implies knowledge
of the criminality of an act. For abetment by instigation, the instigation. must have reference to the
thing that was
done and not to the thing that was likely to have been done by the person who is instigated. It is only if this condition
is
fulfilled that a person can be guilty of abetment by instigation. Under section 107, IPC, it is the instigation of the
commission of the
act itself which constitutes the offence which is regarded as abetment. Instigation does not
merely mean the placing of temptation to
do a forbidden thing, but the actively stimulating a person to do it.
There is no instigation unless it is communicated to the author of
the act in some form or the other. Instigation may
be personal or through a letter or a third party. The act of causing humiliation is not
instigation. The instigation may
be direct or indirect, but in either case it is necessary to show that the mind of the person instigated
was affected
thereby. If the instigation is by means of a letter sent through post, the offence of abetment by instigation is
completed
as soon as the contents of such letter become known to the addressee, and the offence is triable at the
place where such letter is
received. An advice per se cannot necessarily be instigation. Advice can become
instigation only if it is found that it was an advice
which was meant actively to suggest or stimulate the commission
of an offence. In Jai Narain v. State of Bihar, where the persons
who were members of an unlawful assembly set
fire to a hut in consequence of the order of the accused, the Court held that it would
be too unreal to hold that the
persons who set fire to the hut did so irrespective or independently of the order given by the accused.
In another
case, where accused persons went to the house of the deceased, hurled abuses at her, the words used by them
were
highly provocative and stimulating, the deceased as a result committed suicide, the acts of the accused
persons amounted to
instigation and they were held liable to be convicted under section 306 read with section 107,
IPC. A wilful misrepresentation of a
fact amounts to an abetment of an act within the meaning of section 107,
explanation. There has to be a reasonable certainty in
regard to the meaning of the words used by the instigator in
order to determine whether or not there was instigation, but it is not
necessary to prove the actual words used for
the incitement.
Q 86.   D idolized the character Captain America. Once D commented upon a post of the actor of captain America
stating 'Captain,
Do I have your permission to take down enemies and make America great again?'
The actor replied back- 'Go ahead, Make
America great again'. After that D killed two of his neighbors who
were expatriates from Iran. Now the legal representatives of D
want to file a case of abetment on the actor
of Captain America. Decide.

a)  The legal representatives of D can file a suit of abetment as the actor's words to make America great
again led to the murder.

b)  The legal representatives cannot file a suit as it was mere permission which did not intend or aspire to
commit the act.

c)  The actor shall be held liable for abetment as the murder was foreseeable and he was negligent with his
tweets.

d)  The actor shall not be held liable as the acts of replying to a comment will not be considered as an
abetment.

https://www.aspiration.link/LAW/online/QuestionPaper.jsp 24/43
31/10/2022, 15:39 Question Paper

Q 87.   A and B met in a restaurant. A was a former felon. He mentioned to B about Randy Wash, an infamous drug
dealer and
thief who made millions by cheating the government. B was inspired by Randy Wash and attempted
to commit a theft of a bank. He
was caught during his attempt. Can A be held liable for abetment?
a)  A can be held liable for abetment as he knowingly and with intention abetted B to undertake theft.

b)  A cannot be held liable as he did not directly intend to incite or encourage B to commit theft.

c)  A can be held liable as he indirectly affected the mind of B and influenced him to commit theft.

d)  A cannot be held liable as he did not expressly ask or mention B to commit theft.

Q 88.   What did the court hold in the case of Jai Narain v. State of Bihar?
a)  The court held that indirect instigation cannot be a ground for abetment in normal circumstances.

b)  The court held that the act of causing humiliation is not instigation.

c)  
The Court held that it would be too unreal to hold that the persons who set fire to the hut did so irrespective
or independently of the
order given.
d)  None of the above.

Q 89.   A and B were neighbors and always fought with each other. Once it was realized that A's son was in love
with B's daughter.
When A asked for a marriage proposal, B said "you and your family are good for nothing
and you can never get a bride like my
daughter." Enraged by this, A went on to kidnap Twinkle, the most
beautiful girl of the village and tried to marry her to his son. He
was stopped by the police. Now, A wants to file
a case of abetment on B. Decide.

a)  There is no abetment in the given case, as humiliation cannot be considered as an abetment.

b)  B willfully abetted A knowing that he would kidnap twinkle. Hence, there is abetment in the given case.

c)  B cannot be charged with abetment unless he accepts that he knew A would kidnap twinkle.

d)  There is abetment as B's words were an indirect challenge to marry A's son to the most beautiful girl in
the village.

Q 90.   Once the kingdom of Dholakpur was attacked by the kingdom of Pehlwanpur. Bheem, the king of Dholakpur
went to fight in
the war. One Kalia informed the queen Indumati that Bheem had died in the battle and the
troops of Pehlwanpur were breaching
the castle. Only Raju was fighting from their side and everyone else
was dead. Shocked by this and not wanting to be taken as a
prisoner, Indumati committed suicide.
Later it was revealed that Kalia had blatantly lied and Dholakpur had won the battle. Will
Kalia be held liable
for abetment under section 107?
a)  Kalia will not be held liable for abetment as he did instigate Indumati to commit suicide.

b)  Kalia will be held liable as his act of misrepresentation of facts had led to the suicide of Indumati.

c)  Kalia shall not be held liable as he only misrepresented the facts and did not instigate any abetment.

d)  Kalia should be held liable for murder as his statement had led to the death of Indumati.

Directions for questions 66 to 105: You have been given some passages followed by questions based on each
passage. You are
required to choose the most appropriate option which follows from the passage.
Only the information given in the passage should be
used for choosing the answer and no external knowledge of
law howsoever prominent is to be applied.

Passage – 6

Unnatural offences are covered in IPC under section 377. Section 377 of the IPC states that "whoever voluntarily
has carnal
intercourse against the order of nature with any man, woman or animal, shall be punished with
imprisonment for life, or with
imprisonment of either description for a term which may extend to ten years,
and shall be liable to fine". As per the explanation
provided under this section, penetration is sufficient to constitute
carnal intercourse. This section corresponds to the offences of

https://www.aspiration.link/LAW/online/QuestionPaper.jsp 25/43
31/10/2022, 15:39 Question Paper
sodomy and bestiality under the English law.
As evident from the language of this section, consent is wholly immaterial in the case of
unnatural offences and
the party consenting would be equally liable as an abettor. This section is very vague as what is against the
order
of nature is not possible to define objectively. What is natural and what is not is a subject of debate and has led to
much
confusion. As per this section homosexuality is construed as an unnatural offence as it is considered to be
against the order of
nature. This has led to many controversies and has led to questions regarding the constitutional
validity of this section. Thus, in order
to determine the constitutional validity of this section and the reasons for its
incorporation in the IPC it is important to look at its
historical basis.

In Khanu v Emperor it was held that "the natural object of carnal intercourse is that there should be the possibility
of conception of
human beings, which in the case of coitus per so is impossible". The courts in India have interpreted
the term "carnal intercourse
against the order of nature" so broadly that it now includes from oral and anal sex to
penetration into artificial orifices such as folded
palms or between thighs. Such a wide application of section 377
where the language itself is not very clear has led to arbitrary
application of the law and thus questions were raised
regarding the constitutional validity of this section. Apart from this, section 377
clearly makes homosexuality illegal
on the ground that it is against the order of nature. This has also led to various controversies in
view of recognition
of right to freedom as a fundamental human right, it is considered world over that criminalization of homosexual
acts is a clear violation of right to privacy. In view of the arbitrariness of section 377 and violation of basic fundamental
rights, the
constitutional validity of this section was challenged in the court.

The constitutional validity of section 377 was challenged in the Delhi High Court in the case of Naz Foundation v
Government of
Delhi & Ors. In this case it was argued that s 377 on account of covering consensual sexual
intercourse between two adults in
private, is violative of the fundamental rights guaranteed in Articles 14, 15, 19
and 21 of the Constitution. It was also contended that
Article 21 can be curtailed only in case of compelling state
interest, which is missing in this case. The petitioner also contended that
the legislative intent behind section 377
is based on stereotypes that are outmoded and have no historical or logical backing. They
also argued that the
expression "sex" as used in Article 15 also includes "sexual orientation" and thus according to Article 15 there
can
be no discrimination on the basis of sexual orientation. Broadly they prayed before the court that section 377 of IPC
should be
declared ultra vires to the constitution, insofar it criminalizes consensual sexual acts of adults. In this
landmark case, the two-bench
judge held that treating consensual homosexual reating consensual homosexual
sex between adults as a crime is a violation of
fundamental rights protected by India's Constitution
Q 91.   Which of the following was not a ground of pleading for the appellants in the case of Naz Foundation v
Government of Delhi
& Ors?

a)  Section 377 of Indian Penal code is violative of articles 14, 15, 19 and 21 of the constitution.

b)  Article 21 can be curtailed only in case of compelling state interest, which is missing in this case.

c)  
Article 15 also includes "sexual orientation" and thus according to Article 15 there can be no discrimination
on the basis of sexual
orientation.
d)  Section 377 is against the principles held by our founding fore father Jawaharlal Nehru.

Q 92.   Why has there been arbitrary application of law in case of section 377 of IPC?

a)  
Section 377 of IPC uses the term 'against the order of nature' which is in itself quite confusing and
contradictory and has allowed
the courts to apply it in whatsoever manner they choose.

b)  Very broad interpretation of section 377 has led to the arbitrary application of the same.

c)  
As per this section homosexuality is construed as an unnatural offence as it is considered to be against
the order of nature and this
had led to arbitrary rules against LGBTQ+ community.
d)  Right to freedom of fundamental rights have been restricted and the court has bent the restriction according
to their choice.

Q 93.   Ram and Ninan were two adults and were in love with each other. Once they moved into a new place in the
town of
Ramkashi and rented a place. The news of their homosexual relationship spread like wildfire in the
town. One night the localites
broke into their house and found them in a compromising position.
They harassed both of them and handed them over to the
police. This incident happened after the supreme
court judgement of the Naz foundation. Pick the correct answer.

https://www.aspiration.link/LAW/online/QuestionPaper.jsp 26/43
31/10/2022, 15:39 Question Paper

a)  Ram and Ninan shall be liable under section 377 of the Indian penal code 1860.

b)  
Ram and Ninan shall not be protected under the Naz foundation judgment as their act should have been
with resepct to public
order, health and morality.
c)  
Ram and Ninan shall not be held liable as both of them being consenting adults have the right to be ina
relationship and have sex.

d)  Ram and Ninan should have thought before moving into Shivkashi and the local practices need to be
upheld.

Q 94.   Which of the following statements is not an interpretation of section 377 of IPC?

a)  
the natural object of carnal intercourse is that there should be the possibility of conception of human
beings, which in the case of
coitus per so is impossible.

b)  Carnal intercourse is a disgrace on humans and their ability to live in a dignified manner.

c)  
carnal intercourse against the order of nature" includes from oral and anal sex to penetration into artificial
orifices such as folded
palms or between thighs.
d)  
treating consensual homosexual reating consensual homosexual sex between adults as a crime is a
violation of fundamental rights
protected by India's Constitution
 

Q 95.   Which of the following statements is false in the light of the above given passage?
a)  Section 377 corresponds to the offences of sodomy and bestiality under the English law.

b)  
The constitutional validity of section 377 was challenged in the Delhi High Court in the case of Naz
Foundation v Government of
Delhi & Ors.

c)  
In Naz foundation case it was argued that section 377 violates Article 15 which says there can be no
discrimination on the basis of
sexual orientation.

d)  None of the above.

Directions for questions 66 to 105: You have been given some passages followed by questions based on each
passage. You are
required to choose the most appropriate option which follows from the passage.
Only the information given in the passage should be
used for choosing the answer and no external knowledge of
law howsoever prominent is to be applied.

Passage – 7

Change is the only constant. This is also true when it comes to interpretation of statutes. While the fundamental
core of our legal
principles remains universal and classic, their interpretations change and evolve with time.
One such example of evolution could be
the interpretation of the relationship between Article 14, Article 19 & Article
21 under Part III of the Indian Constitution

Article 14, it provides for equality before law and equal protection of the law. It means that no person is deprived of
his equality
among other citizens of our country. The provision also gains importance because the enactment of
such a provision leads to the
abolishing of certain inhuman customary practices of our country. The provisions of
this article also envisage certain legal rights like
protection of law which purely means that the law should be the
same for every person with some necessary exceptions.

According to the Golden Triangle rule established by the infamous Maneka Gandhi v. Union of India, Article 14,
Article 19 & Article
21 are connected with each other. They are complementary to each other and grant the most
basic rights to the Indian citizens and
even non-citizens of India in some cases. Right to equality, right to basic
freedoms and right to life & personal liberty - all must be
read together and collectively interpreted. As per the
Golden Triangle jurisprudence, if someone's personal liberty has been

https://www.aspiration.link/LAW/online/QuestionPaper.jsp 27/43
31/10/2022, 15:39 Question Paper
encroached - it must necessarily pass the
test of violation of all the three articles in order to be categorized as "infringement of
personal liberty".

Article 19 provides certain absolute rights such as freedom of speech and expression, freedom of movement,
freedom of forming
associations and unions, etc. This Article brings about important changes in the society as it
provides various rights to the people so
that there is harmony among the people of our country. Even though this Article covers a vast area of operation, it does not provide a
person the freedom to do anything and everything as
per his whims and fancies. Various other provisions of the Article provide
restrictions to various issues affecting
public tranquillity and security.

Article 21 is applicable even during the time of election wherein people have the sole right of electing the best
person as their
representative. No person has a right to compel anyone to elect the person other than his/her wish.
Even though voting is not a
fundamental right but a 'statutory right', the court, in the judgment of the case PUCL v.
Union of India[8], distinguishes "right to vote"
and the "freedom of voting as the species of the freedom of expression"
under Article 19 of the Constitution. There are various other
major judgments in cases regarding enforcement of
fundamental rights. For example, the case of Kesavananda Bharati v. Union of
India[9], which is considered as a
landmark among cases regarding the enforceability of constitutional rights in favour of the citizens.
The judgment
in the said case makes it clear that even the Central or State Government has certain limitations in encroaching
into a
person's rights, mainly fundamental rights.
Q 96.   What is the basic thumb-rule of doctrine of Golden triangle according to the passage?

a)  
The doctrine of Golden triangle suggests that while applying the said three articles the other has to be
imperatively taken into
consideration as they don't exist without the other.

b)  
According to the golden triangle the three articles are contemporary to each other and grant the most
basic rights to the Indian
citizens and even non-citizens of India in some cases.

c)  Golden triangle is the term used by Dr. BR Ambedkar in constitutional assembly debates in the year
1949.

d)  The golden triangle thumb rule is that article 21 takes the precedence while article 14 comes next followed
by article 19.

Q 97.   How can one categorize "infringement of personal liberty"?


a)  Infringement of personal liberty can be considered after looking into violation of article 21.

b)  
Infringement of personal liberty can be taken into consideration only after taking into consideration all the
three articles of the
golden triangle.

c)  
It must necessarily pass the test of violation of all the three articles in order to be categorized as
"infringement of personal liberty".

d)  Both (b) and (c)

Q 98.   Which of the following statements is false in the light of the above given passage?
a)  
Article 19 provides certain absolute rights such as freedom of speech and expression, freedom of
movement, freedom of forming
associations and unions, etc.
b)  Article 14 provides only for the right to equality before law.

c)  Article 19 is an integral part of the principle of Golden triangle.

d)  The principle of the golden triangle was developed in the case of Maneka Gandhi.

Q 99.   What is the example provided in the passage to suggest the existence of broad application of article 21?
a)  Article 21 extends the right to sleep peacefully as held in the case of Re Rammaidan.

https://www.aspiration.link/LAW/online/QuestionPaper.jsp 28/43
31/10/2022, 15:39 Question Paper

b)  
Article 21 also applies to the right to protest peacefully, the same also is covered under article 19.
This shows the inextricable
nexus between these articles.

c)  
Article 21 is applicable even during the time of election wherein people have the sole right of electing the
best person as their
representative.

d)  The passage does not discuss the broader application of article 21 and constricts itself to the idea of
golden triangle.

Q 100.   What specific idea regarding the interpretation of statues does the author of the passage have?

a)  
The author of the passage strongly suggests that interpretation of the statute should be made in consonance
with the idea of
formalism.
b)  The author of the passage feels that interpretation of statutes changes and evolves with time.

c)  The author does not provide any specific opinion regarding the interpretation of statues.

d)  Both (a) and (b)

Directions for questions 66 to 105: You have been given some passages followed by questions based on each
passage. You are
required to choose the most appropriate option which follows from the passage.
Only the information given in the passage should be
used for choosing the answer and no external knowledge of
law howsoever prominent is to be applied.

Passage – 8

Several states have provisions in their constitutions that mention the environment and (loosely) define the role of
the state
government in protecting the state's natural resources and the health of its residents. One state is
particularly unique in that it
dedicates an entire article of its constitution to natural resources. Article VIII, section 2
of the Alaska State Constitution declares that
"[t]he legislature shall provide for the utilization, development,
and conservation of all natural resources belonging to the State,
including land and waters, for the maximum
benefit of its people." As such, the state's natural resources are held in trust by the
government for the people.

The idea that governments should hold certain natural resources in trust for public use dates to ancient times.
One early example
comes from Byzantine Emperor Justinian, who recognized "public values in water" and ordered
that the sea, the seashores, running
water, and the air were common to everyone. Today, the notion that common
resources should be held in trust for the public-dubbed
the public trust doctrine-is most often applied to shorelines
between low and high tides. Importantly, whether a certain resource falls
under the protection of public trust
involves a state-by-state analysis.

In the late 20th century, the public trust doctrine gave rise to a developing area of environmental law called
"atmospheric trust
litigation." Championed by Professor Mary Christina Wood, this innovative legal theory attempts
to force governments to control
climate change-contributing greenhouse gas (GHG) emissions into the atmosphere.
In perhaps the most famous example of
atmospheric trust litigation, Juliana v. United States, a coalition of youth
plaintiffs brought a lawsuit arguing that the US Constitution
guarantees an unenumerated fundamental right to a
"stable climate system." The deciding court recognized the evidence regarding
the plaintiffs' injuries from climate
change but "reluctantly concluded" that the requested remedies were best addressed by the
political branches of
government: the executive and legislative branches, not the judiciary.

Since the 1970s, political branches in five states-Pennsylvania, Montana, Illinois, Massachusetts, and Hawaiihave
enacted
constitutionally based environmental rights provisions, including the right to a "clean and healthful
environment." Pennsylvania was
first in 1971, followed by Montana in 1972. The Rhode Island Constitution also
includes specific environmental protections in its 1987
bill of rights; however, those protections only extend to
fishery rights and shore privileges, and therefore the state is not generally
regarded as having a green amendment.

Proposing Constitutional Amendments


There are four primary methods of passing proposing constitutional amendments for ratification. Many state
constitutions allow for
multiple methods. However, two have been successful means of proposing and passing
green amendments:
https://www.aspiration.link/LAW/online/QuestionPaper.jsp 29/43
31/10/2022, 15:39 Question Paper
• Through legislatively referred constitutional amendments;
• Through citizen-initiated constitutional amendments;
• Through constitutional conventions; and
• Through commission-referred constitutional amendment.

Furthermore, every state except Delaware requires voters to ratify proposed constitutional amendments.
Q 101.   What idea does the Article VIII, section 2 of the Alaska State Constitution give?
a)  Article VIII, section 2 gives autonomy over the use of natural resources to the government.

b)  Article VIII, section 2 gives the idea that governments should hold certain natural resources in trust for
public use.

c)  Article VIII, section 2 bans illegal destruction of forests and other protected areas,

d)  Article VIII, section 2 provides safeguarding duties of the government to protect natural resources.

Q 102.   How has the atmospheric trust litigation proved to be successful or can be claimed as an effective remedy?

a)  The atmospheric trust litigation can be claimed as an effective remedy as its genesis is from public trust
doctrine.

b)  
Public trust doctrine has been previously held successful and hence the atmospheric trust litigation can
also be considered to be
successful.

c)  
The case of Juliana v. United States for the first time showed that atmospheric trust doctrine can be
successful and is an effective
remedy.
d)  None of the above.

Q 103.   Which of the following statements is false in the light of the above given passage?
a)  
Since the 1970s, political branches in five states of the USA have enacted constitutionally based
environmental rights provisions.

b)  
Today, the notion that common resources should be held in trust for the public is most often applied to
shorelines between low and
high tides.

c)  
The idea that governments should hold certain natural resources in trust for public use dates to ancient
times. One early example
comes from Byzantine Emperor Justinian

d)  None of the above.

Q 104.   Why is the Rhode-island constitutional provision of the environment different from others?

a)  Rhode-island provision is much constricted and strict while considering the other state provisions.

b)  
Rhode-island protections only extend to fishery rights and shore privileges, and therefore the state is not
generally regarded as
having a green amendment.
c)  
Rhode-island protections only extend to marine and sea shores and therefore the state is not generally
regarded as having a green
amendment.
d)  None of the above.

Q 105.   Why does the passage hail Alaska State Constitution by comparing it with the rest of the other state
constitutions?

a)  The Alaska state constitution has been more vigilant and has brought in better rules to safeguard the
greenery.

https://www.aspiration.link/LAW/online/QuestionPaper.jsp 30/43
31/10/2022, 15:39 Question Paper

b)  The passage does not provide the definite reason for the same.

c)  The Alaska state constitution has adopted the public trust doctrine.

d)  Both (a) and (c).

Directions for questions 106 to 135: Read the passages and answer the questions that follow.

Passage – 1

Ominous clouds hover over India's "Sunshine Legislation", enacted with great fanfare in 2005. Amidst renewed
concern over its
functioning across states, the Right to Information Act (RTI) is set to complete 17 years this
October. A damning indictment of the
various Information Commissions - "acting as a major bottleneck in effective
implementation of the RTI Law" - comes from the report
of the Sarthak Nagrik Sangathan (SNS) 2021.

A huge backlog of second appeals, lengthy wait time for hearings, hesitancy in posting penalties and increasing
opacity in the
working of the commissions have led SNS to this conclusion. The commissions have been plagued
with vacancies, poor choice of
commissioners, untrained staff and a non-cooperative set of public information
officers (PIOs). The sufferers, however, are the
applicants.

The RTI Act is a sunshine legislation aimed at eradicating corruption and promoting transparency. The Indian
information law, rated
as one of the strongest in the world, needs to be bolstered by raising awareness amongst
the people and organising rigorous
training of government officials. A strong political system is a must for the RTI
regime to flourish.

It is imperative to ensure freedom of the press and democratic institutions, punish errant officials and maintain
complete autonomy of
the information commissions, in the interest of the people and the nation at large.
As India emerges as a global power, the
implementation of legislation like the RTI Act will be under the constant
scrutiny of the comity of nations.
Q 106.   Which of the following is the overall conclusion of the whole passage?

a)  RTI Act should be revamped as it is completing 17 years this October, new changes must be introduced.

b)  There has been a wide range of corruption being observed when it comes to Right to Information Act,
this has resulted in RTI.

c)  
It has now become very necessary to give freedom to the information commissions so that the needs of
the people could be dealt
with at a larger level without any corruption.
d)  In the coming years, RTI will be under scrutiny by a majority of countries.

Q 107.   Which of the following expresses the paradox of the passage?

a)  
RTI act being right to information Act is in fact concealing the information from people instead of giving
them appropriate
information.

b)  Despite completing 17 years in October, RTI Act has been ineffective in dealing with corruption along
with lack of awareness.

c)  RTI Act is considered to be a sunshine legislation, however it is still filled with deep cracks of corruption
and lack of freedom

d)  The applicants are the sufferers and not the one who have legislated this act.

Q 108.   Which of the following inferences CAN BE DRAWN from the passage?

a)  
A huge backlog of appeals, lengthy wait time for hearings, hesitancy in posting penalties and increasing
opacity in the working of
the commissions have led SNS to this conclusion.

b)  
A damning indictment of the various Information Commissions - "acting as a major bottleneck in effective
implementation of the RTI
Law" - comes from the report of the Sarthak Nagrik Sangathan (SNS) 2022.

https://www.aspiration.link/LAW/online/QuestionPaper.jsp 31/43
31/10/2022, 15:39 Question Paper

c)  
Ominous clouds hover over India's "Sunshine Legislation", enacted with great fanfare in 2005.
Amidst renewed concern over its
functioning across states, the Right to Information Act (RTI) is set to
complete 18 years this October.

d)  
It is important to ensure freedom of the press and democratic institutions, punish errant officials and
maintain complete autonomy
of the information commissions.

Q 109.   Which of the following WEAKENS the passage the most?

a)  Being a sunshine legislation, RTI envisages equal right to information to all even though it is being corrupted
on many levels.

b)  Errant officials must be punished and brought to court for justice to be served

c)  Other countries do not have a locus standi to interfere in the RTI Act of India.

d)  RTI Act would further improve if it will be under a constant scrutiny by the nations.

Q 110.   Which of the following STRENGTHENS the passage the most?


a)  RTI Act is going to be further improved in future considering the breakthroughs that have occurred in the
recent years

b)  
The Supreme Court has also emphasised the importance of the Right to Information Act and has
recommended various measures
in order to improve it.

c)  Both (a) and (b) are correct answers.

d)  neither (a) nor (b) are correct answers.

Directions for questions 106 to 135: Read the passages and answer the questions that follow.

Passage – 2

A virus belonging to the poxvirus's family causes a rare contagious rash illness known as monkeypox. This zoonotic
viral disease (a
disease transmitted from animals to humans) has hosts that include rodents and primates.
What was previously limited to the
disease's local spread in central and west Africa, close to tropical rainforests,
has recently been seen in various urban areas and
now in more than 50 countries. The good news is that most
infected people will have minor illnesses and recover on their own.
Despite mild illness and a low transmission
rate, the WHO has declared monkeypox, a public health emergency of international
concern (PHEIC) to contain
the disease. The increase in monkeypox cases in a short span of time in many countries necessitated
the declaration
of PHEIC and additional research studies. It is unclear whether the recent sudden outbreaks in multiple countries
result from mutations that alter virus transmissibility. SARS-CoV-2 and monkeypox virus co-infection can alter
infectivity patterns and
response to vaccination against both diseases. As a result, there is a need to improve test
efficiency.

The world is yet to recognise emerging infectious diseases as a threat. The immediate priority is to strengthen the
surveillance
infrastructure, including hiring public health professionals who can participate in outbreak detection
during many future PHEICs.
Mechanisms for initiating contact tracing, quarantining exposed people should be
institutionalised. Without prioritising public health
strengthening, the threat of new and re-emerging infectious
diseases, as well as the enormous social and economic challenges that
accompany them, is real and grave.
Q 111.   Which of the following is the main point of the passage?

a)  The world should recognise monkeypox as a disease which could threaten the public health systems of
various countries.

b)  
The priority of each country now should be to improve their health infrastructure. Challenges such as
social and economic must be
averted.
c)  We have grave public health challenges before us, they must be averted before any serious emergency
arises

https://www.aspiration.link/LAW/online/QuestionPaper.jsp 32/43
31/10/2022, 15:39 Question Paper

d)  
Monkeypox is a virus quite similar to SARS COV-2 and vaccination against either disease is not fully
available. An alternative must
be searched to combat these diseases.
 

Q 112.   Which of the following is the paradox drawn from the passage?
a)  
Despite mild illness and a low transmission rate, the WHO has declared monkeypox, a public health
emergency of international
concern (PHEIC) to contain the disease.
b)  Though monkeypox is a transmissible disease still the countries are not taking action to tackle the disease
properly.

c)  
COVID-19 spread very rapidly and the countries were reluctant to take any action at first, similarly the
same should not be done
with monkeypox as then the situation would be quite paradoxical.

d)  
Going through COVID-19, our infrastructure should have been robust as of now to tackle monkeypox,
however it has become more
weakened instead of strengthening.

Q 113.   Which of the following inferences cannot be drawn from the passage?

a)  This zoonotic viral disease (a disease transmitted from humans to animals) has hosts that include
rodents and primates.

b)  
Without prioritising public health strengthening, the threat of new and re-emerging infectious diseases,
as well as the enormous
social and economic challenges that accompany them, is real and grave.

c)  
The good news is that most infected people will have minor illnesses and recover on their own.
Despite mild illness and a low
transmission rate, the WHO has declared monkeypox, a public health
emergency of international concern (PHEIC) to contain the
disease.

d)  SARS-CoV-2 and monkeypox virus co-infection can alter infectivity patterns and response to vaccination
against either disease.

Q 114.   Which of the following weakens the passage the most?


a)  
Monkeypox should not be treated as a disease of serious nature, we have encountered many diseases
like this before and are well
equipped to handle this as well.
b)  
Our infrastructure has become more strengthened from the past encounter with the diseases we have,
therefore efforts should be
channelised by using them to prevent other threatening diseases.
c)  Both (a) and (b) weaken the passage.

d)  Neither (a) nor (b) weakens the passage.

Q 115.   "The world is yet to recognise emerging infectious diseases as a threat". Which of the following purposes
this line serves?

a)  It acts as a conclusion of the passage.

b)  It acts as a premise to the passage.

c)  It acts as an assumption to the passage.

d)  It acts as a paradox to the passage.

https://www.aspiration.link/LAW/online/QuestionPaper.jsp 33/43
31/10/2022, 15:39 Question Paper
Directions for questions 106 to 135: Read the passages and answer the questions that follow.

Passage – 3

Another Dozen Stories, by Satyajit Ray, translated from the Bengali by Indrani Majumdar. The book features
never-before translated
stories of spooky, magical and astonishing worlds created by the polymath. We spoke
with Majumdar about these Ray's world of
fascinating stories. On asking Indrani to tell us about her experience
while translating the stories included in the book, she gave us
this answer.

"Ray's style of writing at one level is very simple, colloquial and down to earth. Yet, this style posed a huge
challenge to me. The
opening story in this volume, The Life and Death of Aryasekhar, written way back in 1968, is
one that Ray labelled as 'the only story
written with adults in mind'. Not just the theme, what was a complete
departure was the use of a very strict, formal Bengali style of
writing by Ray. Reading this story, one was more
than impressed, and it revealed Ray's strong hold over his mother tongue. And
here, I put in real effort and labour
over my translation. If readers like reading this book, the credit goes strictly to the writer. Though
translating the
other stories meant for young adults proved less daunting, it meant reworking on the structure of certain sentences
quite a few times. In Bengali, Ray on occasions wrote a sentence so long, it almost amounted to one paragraph.
In the original, it
works rather well. But not necessarily so when transforming it into another language. To ease the
flow of reading, I cut it down to 3-4
sentences to retain the essence. Translating his quaint use of humour and wit
is no less a challenge.

Time and again a twist in the last sentence of a story could never retain the same charm or offer a tone of the same
surprise while
writing in English. I felt translating a language often meant translating a culture, too. For instance,
a story ends with a mere sentence
stating that the protagonist chose to die in the early hours on the day of his
niece's wedding. It imports a note of a huge tragedy as a
death in a Bengali family meant no auspicious activities
to be observed for the next whole year. I'm sure such a custom works
elsewhere, too in India."
Q 116.   According to the passage, which of the following inferences can be drawn about Indrani Majumdar?
I. She is a brilliant and wonderfully successful writer.
II. She, as a translator, adds creativity to her work.
III. She has a deep insight about translating, not just words but emotions too.
IV. She likes getting credit for her work.

a)  Only I

b)  Only II

c)  Both II and III

d)  Both I and IV

Q 117.   "Time and again, a twist in the last sentence of a story could never retain the same charm or offer a tone of
the same
surprise while writing in English."
Which of the following, if true, seriously weakens the argument above?
I. English as a language lacks the power of expression.
II. Bengali is better than English.
III. Satyajit's stories had predictable endings.
IV. The English novels have immense power to hold the interest of the audience.
a)  Only I

b)  Both II and IV

c)  Both III and IV

d)  All I, II, III and IV

https://www.aspiration.link/LAW/online/QuestionPaper.jsp 34/43
31/10/2022, 15:39 Question Paper

Q 118.   "Ray's style of writing at one level is very simple, colloquial and down to earth. Yet, this style posed a huge
challenge to
me."
The above statement makes which of the following assumptions?
I. Ray's writing style is unmatchable.
II. The translator likes taking up challenges
III. Translation demands retaining the essence of the original work

a)  Only I

b)  Only II

c)  Both I and II

d)  Both II and III

Q 119.   Which of the following statement(s) is/are true according to the passage?

a)  It is difficult to translate books written with adults in mind

b)  It is challenging to recreate instances and emotions like the original in the translated version.

c)  Working on sentence structures is the most daunting task of all.

d)  All of the above

Q 120.   Which of the following conveys the main idea expressed in the passage?
a)  Exaggerating the work of Satyajit Ray.

b)  Self-appreciating the efforts of Indrani Majumdar.

c)  Outlining the challenges of a translator.

d)  Comparing English and Bengali.

Directions for questions 106 to 135: Read the passages and answer the questions that follow.

Passage – 4

Human beings are self-conscious creatures: we can conceptualise ourselves as psychological beings, forming
beliefs about who and
what we are. We also have identities: self-beliefs that are sources of meaning, purpose and
value, and that help to constrain our
choices and actions. In addition to being able to think about ourselves,
self-conscious beings can recognise that we are the objects
of other people's thoughts. This opens up the possibility
of a conflict between our own identities and how we are perceived by others.
This potential for conflict gives us
unique power over each other, and also makes us uniquely vulnerable: only self-conscious beings
can kill with a
glance or die of embarrassment. Our vulnerability to how others regard us might create obligations to try to regard
others in some of the ways they desire - ways that are consonant with their own identities. But what about identities
that we think are
false or absurd - or that we simply don't understand?

A plural is a human being who says things like: 'I'm one of many people inside my head.' Although they are quite
rare (it's impossible
to say now rare), plurals are increasingly visible on social media and in the occasional popular
media article. At present, there is a
handbook online about now to respond to a co-worker's 'coming out'
(as the document puts it) as plural. You might think you've
heard of plurals if you've heard of dissociative identity
disorder (DID), because, like plurals, people With DID experience themselves
as being psychologically multiple.
But many plurals don't meet the diagnostic criteria for DID. Often, this is because they don't find
their plurality per
se to be distressing or impairing. In other cases, it's because they don't meet the amnesia criterion for DID,
since
the multiple beings that plurals experience as being inside them can share experiences or communicate to
each other about their
experiences.
Q 121.   Which of the following can we infer about 'plurals' from the given passage?

a)  Plurals might be mistaken for people with dissociative identity disorder.

b)  A plural human being isn't a person at all.

https://www.aspiration.link/LAW/online/QuestionPaper.jsp 35/43
31/10/2022, 15:39 Question Paper

c)  There are identities that we shouldn't respect, because they reinforce unjust social arrangements.

d)  Plurals live with a discord between what they believe about themselves and what everyone else believes.

Q 122.   We can infer from the passage that…

a)  Plurality is the by-product of the online social media platforms.

b)  Human beings can counter conflicts by becoming impervious to others perception about self.

c)  What we can learn about respect and identity from plurals.

d)  Human beings have many mental malaises that they are unaware of.

Q 123.   Which of the following is most likely to be true had human beings not cared about what others think of them?
a)  Human beings would be facing existential crisis.

b)  The element or concept of plurality might not be in existence at all.

c)  There won't be any vulnerability among human beings.

d)  There won't be any conflict among human beings.

Q 124.   Which of the following forms the premise for the author's argument that potential for conflict makes us
powerful and
vulnerable at the same time?
a)  
By virtue of possessing the element of plurality, we can experience two contrasting emotions (powerfulness
and vulnerability) at the
same time.
b)  Human beings experience an identity crisis if they don't feel multiple emotions all at once.

c)  
We can form opinions about others which makes us powerful and at the same time will be judged by
others, making us vulnerable.
d)  Responding to a particular situation requires a mixed set of emotions.

Q 125.   Which of the following, if true, would weaken the argument made in the passage that we are objects of other
people's
thoughts?
a)  
Out in the social world - that is, the social world outside their heads - plurals mostly live as if they were the
way that singlets see
them.
b)  It generally matters to us that other people respect those identities which matter to us.

c)  
A relationship with plurals would put one in a position to grasp what their plural identities mean to them -
what this does for them,
what it scaffolds or supports in their lives.

d)  None of the above

Directions for questions 106 to 135: Read the passages and answer the questions that follow.

Passage – 5

J F C Fuller did not invent the tank.


That distinction should probably fall to E L de Mole, an Australian who approached the British war office in 1912
with a design that
was - in the words of historians Kenneth Macksey and John Batchelor - "so convincingly similar
to those which finally went into
service that one wonders why it was never adopted from the outset". But when the
British army eventually introduced the tank, it was
https://www.aspiration.link/LAW/online/QuestionPaper.jsp 36/43
31/10/2022, 15:39 Question Paper
J F C Fuller, chief staff officer of what would later become the
tank corps, who understood what to do with it. Late in 1917, after
almost 400 British tanks had, with modest
success, lumbered across the German lines at the battle of Cambrai, Fuller applied his
radical streak to the
problem of using the tank effectively. A Medium D could roll across the trenches and be on the German
command
posts in an hour; Fuller's attack would come from nowhere. Air support would disrupt German road and rail travel.
"Bad
news confuses, confusion stimulates panic," wrote Fuller. By striking suddenly at the German command,
Plan 1919 would cause the
German army to disintegrate. It would, Fuller declared, be "the winning of the war in a
single battle".

His astonishing idea became "the most famous unused plan in military history", according to his biographer Brian
Holden Reid. But,
of course, that is not entirely true. It was used to great effect, in 1940 - by the Germans.
J F C Fuller had invented blitzkrieg.

The story might be a historical curiosity, had echoes of it not been repeated so frequently since the British army
stuffed Fuller's plans
for blitzkrieg into a desk drawer. Organisations from newspapers to oil majors to computing
giants have persistently struggled to
embrace new technological opportunities, or recognise new technological
threats, even when the threats are mortal or the
opportunities are golden.

In 1970, the photocopying giant Xerox established the Palo Alto Research Center, or Parc. Xerox Parc then developed
the world's
first personal computer, with a graphical user interface, windows, icons and a mouse. Bill Gates of
Microsoft and Steve Jobs of Apple
observed developments at Xerox Parc with great interest. Xerox still makes
photocopiers.
Q 126.   J F C Fuller did not invent the tank. Through the statement, the author does which of the following?

a)  The author clears the disseminated misconception

b)  The author categorically puts aside a common perception.

c)  The author draws the attention of the readers towards the truth.

d)  The author expresses regret for the misconception around who invented the tank.

Q 127.   Which of the following situations is parallel to the reasoning provided in the passage?
a)  
IBM - the giant of mainframe computing. IBM is a survivor. It predates the digital computer by more than
three decades. While the
performance of computers was being revolutionised by the semiconductor,
the integrated circuit, the hard drive and the compiler,
IBM maintained a dominant position without breaking
stride.
b)  Unsuccessful organisations stick to their once-triumphant strategies, even as the world changes around
them.

c)  
When Steve Jobs visited Xerox Parc in 1979, and saw a windows-and-mouse interface for the first time,
he couldn't contain
himself, according to an article by Malcolm Gladwell. "Why aren't you doing anything
with this?" he yelled. "This is the greatest
thing. This is revolutionary!"
d)  
In 1999, Sony launched the "Memory Stick Walkman", one of the world's first digital music players.
Sony was armed with the iconic
Walkman brand, some of the world's best consumer electronics engineers
and the talent-soaked Sony-BMG music label. The
Memory Stick Walkman went nowhere and, two years
later, it was eclipsed by a similar product that transformed the fortunes of a
struggling Apple: the iPod.
 

Q 128.   The arguments presented in the passage can be the answer to which of the following concerns raised by
the author in
question form?

a)  Why do ideas slip out of the hands of the executors and fall into the hands of the innovators?

b)  What were the reasons behind J F C Fuller's unused radical ideas?

c)  Why do some ideas slip out of the grasp of incumbents, then thrive in the hands of upstarts?

d)  Why do brilliant ideas never find the daylight and are lost in the darkness?

Q 129.   The historical events shared by the author are

https://www.aspiration.link/LAW/online/QuestionPaper.jsp 37/43
31/10/2022, 15:39 Question Paper

a)  anecdotes to show how people miss out on golden opportunities.

b)  to present that a brilliant idea lost by one becomes a golden goose for the other.

c)  an illustration to prove the mistakes made by Xerox Parc.

d)  to show how Germany used the radical idea by JFC Fuller.

Q 130.   What might be the possible explanations to why organisations squander good ideas?
1. Firms fail because they stick to idea(s) that made them successful,"
2. New technologies are flawed or under-developed at first, so do not appeal to their conceptualizers.
a)  Only 1

b)  Only 2

c)  Both 1 and 2.

d)  Neither 1 nor 2.

Directions for questions 106 to 135: Read the passages and answer the questions that follow.

Passage – 6

The French mathematician Pierre-Simon Laplace (1749-1827) believed that the Universe was a piece of machinery,
and that physics
determines everything. Napoleon, who had read up on Laplace's work, confronted him about the
conspicuous absence of a creator
in his theory. 'I had no need of that hypothesis,' came the reply. Laplace might
have said the same thing about free will, which his
mechanistic universe rendered superfluous.

Since Laplace's day, scientists, philosophers and even neuroscientists have followed his lead in denying the
possibility of free will.
This reflects a widespread belief among theoretical physicists that if you know the initial
values of the variables that characterise a
physical system, together with the equations that explain how these
variables change over time, then you can calculate the state of
the system at all later times. For example, if you
know the positions and velocities of all the particles that make up a gas in a
container, you can determine the
positions and velocities of all those particles at all later times. This means that there should be no
freedom for any
deviation from this physically determined trajectory.

Consider, then, that everything we see around us - rocks and planets, frogs and trees, your body and brain -
is made up of nothing
but protons, electrons and neutrons put together in very complex ways. In the case of your
body, they make many kinds of cells; in
turn, these cells make tissues, such as muscle and skin; these tissues
make systems, such as the heart, lungs and brain; and these
systems make the body as a whole. It might seem
that everything that's happening at the higher, 'emergent' levels should be
uniquely determined by the physics
operating beneath them. This would mean that the thoughts you're having at this very moment
were predetermined
at the start of the Universe, based on the values of the particle physics variables at that time.
Q 131.   Which of the following statements may have formed the premise for Laplace's argument that 'I had no need
of that
hypothesis'?
a)  He was an atheist and did not believe in the existence of a sole creator, after all.

b)  He proved that biological activity at the micro level is literally grounded in the physical shape of biological
molecules.

c)  
He might have proved or formulated the existence of all events in the Universe on the basis of physics
and mathematical
equations, rejecting the plausibility of free will.
d)  He failed to explain some of the events which were beyond the realms and reach of theories of physics.

Q 132.   Which of the following would undermine the argument made in the passage that if you know the initial values
of the
variables that characterise a physical system, together with the equations that explain how these
variables change over time, then
you can calculate the state of the system at all later times?

a)  Ions are atoms that have become electrically charged because they have lost or gained an electron.

https://www.aspiration.link/LAW/online/QuestionPaper.jsp 38/43
31/10/2022, 15:39 Question Paper

b)  
Heisenberg's uncertainty principle introduces an unavoidable fuzziness and an irreducible uncertainty in
quantum outcomes. You
might know the value of one variable, such as a particle's momentum;
that means you can't accurately detect another, such as its
position.

c)  
An apple is suspended from a branch of the tree by a string attached to its stalk. It would thereby have
been turned into a
pendulum, because the string constrained its motion. Instead of dropping to the
ground, it would have swung back and forth in a
circular arc under the branch, with its state of motion
determined uniquely by its initial position and velocity.
d)  All of the above.

Q 133.   Which of the following can we infer from the passage above?

a)  
The state of a system is described by what's known as its wave function, which determines the probabilities
of different outcomes
when events take place.

b)  At very small scales, the theory of Physics underlies what's happening in the world.

c)  The structure of the molecules is truly the secret of life.

d)  Outcomes don't depend only on the equations and the initial data.

Q 134.   Which of the following is most likely to be true had theories of physics and mathematical equations failed to
explain some
of the earthly events?
a)  Learning and memory offer examples of how downward causal effects shape the underlying physics.

b)  
If you seriously believe that fundamental forces leave no space for free will, then it's impossible for us to
genuinely make choices
as moral beings.

c)  
The power of choice enables physiological systems such as the heart and brain to function in a way that
is enslaved by the lower-
level interactions, and choosing the outcomes of the preferred interactions from
a multitude of options have been predetermined at
the elementary level.

d)  None of the above.

Q 135.   Which of the following most accurately expresses the main point of the passage?

a)  
The confounding thing for free-will sceptics is that all outcomes don't depend only on the equations and
the initial data; they also
depend on randomization.

b)  In a physically determined world, there is freedom for deviation up to a certain extent.

c)  There are events or things which Physics is unable to answer or reach to an acceptable conclusion.

d)  An understanding of physics sees determinism at work in the universe.

Directions for questions 136 to 140: Answer the questions on the basis of the information given below.

In a factory there are a total of 240 management employees and 400 workers. 65% of the number of management
employees are
officers and the remaining are executives. Out of the total number of workers 45% are females.
20% of the female workers are only
graduates. The remaining female workers are graduates as well as certified
technicians. Three-fourths of the male workers are only
graduates whereas the remaining male workers are
graduates as well as certified technicians. Two-thirds of the officers are males.
Five-fourteenth of the number of
the executives are females.
Q 136.   What is the ratio of the total number of officers to the number of male workers who are graduates only?

https://www.aspiration.link/LAW/online/QuestionPaper.jsp 39/43
31/10/2022, 15:39 Question Paper

a)  13 : 55

b)  55 : 54

c)  11 : 52

d)  52 : 55

Q 137.   What is the total number of male officers, female executives and female workers who are graduates as well
as certified
technicians?
a)  278

b)  315

c)  197

d)  257

Q 138.   The workers who are graduates as well as technicians form approximately what percentage of the total
employees of the
factory?
a)  27%

b)  31%

c)  39%

d)  19%

Q 139.   What is the difference between the total number of males and the total number of females in the factory?

a)  96

b)  132

c)  116

d)  143

Q 140.   All the workers who are graduates only are fresh recruits and they are provided with an induction training
program, which
costs the company Rs.1,250 per head. Find the cost of providing training to all the workers
who are only graduates?

a)  Rs.2,45,250

b)  Rs.1,75,250

c)  Rs.2,90,000

d)  Rs.2,51,250

Directions for questions 141 to 145: Answer the questions on the basis of the information given below.

The table given below shows the percentage distribution of energy released by different energy components and
the total energy
released (in kilocalorie) by combustion of one kilogram of that food product.

https://www.aspiration.link/LAW/online/QuestionPaper.jsp 40/43
31/10/2022, 15:39 Question Paper

It is also given that:


On dozen eggs = One kilogram
Half dozen banana = Half dozen apple = One kilogram
Q 141.   Energy released by combustion of glucose in one kilogram of product, is highest for which of the following
products?
a)  Sprout

b)  Egg

c)  Meat

d)  Fish

Q 142.   If the breakfast diet of a person consists of 250 gm of sprout, one banana and one egg, then what is the total
energy (in
kilocalorie) intake of the person?

a)  1450

b)  1200

c)  1080

d)  1180

Q 143.   For which of the following two products, energy released by combustion of protein in one kilogram of the
product is the
same?

a)  Banana and Apple

b)  Sprout and Fish

c)  Egg and Meat

d)  Sprout and Apple

Q 144.   Energy released by the combustion of iron, in 100 gm each of meat and fish, is approximately what percent
of the total
energy released?

a)  12.86%

b)  11.36%

c)  10%

d)  14.2%

Q 145.   Energy released by combustion of fat in one kilogram of product, is lowest for which of the following products?

https://www.aspiration.link/LAW/online/QuestionPaper.jsp 41/43
31/10/2022, 15:39 Question Paper

a)  Sprout

b)  Egg

c)  Meat

d)  Fish

Directions for questions 146 to 150: Answer the questions on the basis of the information given below.

Three pieces of wire of equal lengths are used to form three different shapes. One piece of wire is bent in the form
of a circle having
radius 42 cm. Another length of wire is bent and made into a square. The third piece is bent to
form an equilateral triangle.
Q 146.   Find the absolute difference in the area of the square and the circle.

a)  1262 cm2

b)  1188 cm2

c)  1328 cm2

d)  1022 cm2

Q 147.   What is the area of the triangle so formed?

a)  

b)  

c)  

d)  

Q 148.   What is the total length of all the three wires?

a)  264 cm

b)  792 cm

c)  692 cm

d)  882 cm

Q 149.   What is the length of the diagonal of a rectangle having length equal to the side of the triangle and breadth
equal to the
side of the square?

a)  110 cm

b)  120 cm

c)  111 cm

d)  121 cm

Q 150.   What is the area of the circle formed by joining all the three wires without overlapping?

a)  440506 cm2

b)  55594 cm2

c)  49896 cm2

https://www.aspiration.link/LAW/online/QuestionPaper.jsp 42/43
31/10/2022, 15:39 Question Paper

d)  39424 cm2

https://www.aspiration.link/LAW/online/QuestionPaper.jsp 43/43

You might also like